You are on page 1of 117

Prof.

Adel Kamel 1

Contents

1-SPINAL CORD 2

External morphology of the spinal cord 2

Internal morphology of the spinal cord 4

Tracts of the spinal cord 6

2-BRAIN STEM 26

Medulla Oblongata 26
Pons 31
Midbrain 34
Cranial nerve nuclei 38
Deep origin of cranial nerve nuclei 40
Reticular Formation 50
3-CEREBELLUM 53
4-SPECIAL SENSORY PATHWAYS 62
Olfactory pathway 62
Visual pathway 64
Hearing pathway 70
Taste pathway 73

5-DIENCEPHALON 76
Epithalamus 76
Subthalamus 77
Thalamus 79
Hypothalamus 91

6-BASAL GANGLIA 99

7-LIMBIC SYSTEM 108


8-COMPREHENSIVE QUESTIONS 115
Prof.Adel Kamel 2

CHAPTER 1

SPINAL CORD
Objectives
-Orientation of the external features, gray and white mater of the spinal cord.
-Familiarity with ascending sensory modalities.
-Concept of descending pyramidal and extrapyramidal tracts
-The anatomical basis of manifestations due to lesions of the spinal cord

EXTERNAL MORPHOLOGY OF THE SPINAL CORD


The adult human spinal cord is 45 cm long and 30 gm in weight. It is nearly cylindrical
structure with a diameter of 1-1.5 cm. Its shape varies from oval to round. Its maximum
circumference is 38 mm at level of C6. It has a cervical (C5-T1) and lumbar (L1-S2)
enlargements for the supply of the upper and lower limbs. The spinal cord occupies the
vertebral canal which provides support and protection for it. It extends from the foramen
magnum where it is continous with the medulla oblongata to the level of the lower border of
L1. The lower end of the spinal cord is tapering cone-shaped and is termed conus medullaris
(involves segments from S3 to Co).

External longitudinal grooves of the spinal cord (fig.1-1)


1-Shallow dorsal median sulcus and a deep ventral median fissure (3 mm deep and lodges
the anterior spinal artery) mark the sagittal plane of the spinal cord.
2-A shallow dorsolateral and ventrolateral sulci mark the line of attachments of the dorsal
and ventral nerve roots.
3-A shallow dorsal intermediate sulcus is found between gracile and cuneate tracts.
Suspending factors
1-Filum terminale is a pial filament which extends from the tip of the conus medullaris,
pierces the dural sac, and passes the sacral hiatus to fuse with the back of the first coccygeal
vertebra.
2-Denticulate ligaments are two transverse bands that extend from the pia mater to attach to
dura mater.
3-Spinal nerve roots are the main supporting factor.
Prof.Adel Kamel 3

Spinal nerves
31 pairs of spinal nerves emerge from the spinal cord (8C, 12T, 5L, 5S and 1 Co), by two
roots; dorsal (afferent or sensory) and ventral (efferent or motor). The dorsal and ventral
roots join to pass to their corresponding intervertebral formen. Dorsal root ganglia appear as
small enlargements on the dorsal roots near their union with the ventral roots at the
enterance to the intervertebral foramina. Below cervical segments, spinal nerves follow an
increasingly oblique course to reach their corresponding intervertebral formina. This is most
marked for the lumbosacral nerve roots which descend below the termination of the spinal
cord in horse-tail fashion termed cauda equine.

Immediately after leaving the intervertebral foramina, spinal nerves divide into dorsal ramus
and ventral ramus. The dorsal ramus supplies muscles and skin of the back. The ventral
ramus supplies muscles and skin of the front region and limbs.

Diagram of a typical spinal nerve.


Prof.Adel Kamel 4

INTERNAL MORPHOLOGY OF SPINAL CORD


The spinal cord consists of central gray matter and peripheral white matter.
Gray matter of the spinal cord
Gray matter is H-shaped and centrally located. Neurons of the gray matter are organized
into nuclei and laminae. The gray matter is subdivided into sensory portion, the posterior
horn, and a motor portion, the anterior horn, separated by intermediate zone. The sensory
portion includes sensory relay neurons which receive posterior root input and whose axons
contribute to ascending pathways. The motor portion has motor neurons whose axons exit in
the anterior roots. However 90% of the spinal neurons are interneurons whose axons do not
leave the spinal cord. Neurons of gray matter are organized in clusters called nuclei. These
nuclei may extend for a distance or for whole length of the spinal cord, forming columns of
functionally related cells.

Fig.1-1.External and internal features of the spinal cord.


Prof.Adel Kamel 5

White matter of the spinal cord

The white mater of the spinal cord is subdivided into posterior, lateral and anterior
funiculi, demarcated by sulci and dorsal and ventral root entries (fig.1-2). Axons in the white
mater form pathways that are classified as ascending and descending tracts. Individual tracts
run in specific funiculi of the spinal cord.

Fig.1-2. Tracts of the spinal cord.


Prof.Adel Kamel 6

TRACTS OF THE SPINAL CORD

A tract is a group of nerve fibers which have the same origin, the same destination, and
subserve the same function. Tracts of the spinal cord are either short or long. Short tracts
ascend or descend for short distances and end in the spinal cord (spino-spinal tracts). Long
tracts are also ascending and descending, but they have more than one relay station in their
pathways and end or begin at a higher center than the spinal cord.

Long tracts of spinal cord

Ascending tracts Descending tracts

I-Conscious sensory tracts I-Pyramidal tract


1-Gracile and cuneate tracts 1-Corticobulbar tract
2-Lateral spinothalamic tract 2-Corticospinal tract
3-Anterior spinothalamic tract

II-Extrapyramidal tracts
II-Unconscious informative 1-Rubrospinal tract

1-Spinocerebellar tracts 2-Vestibulospinal tract


2-Spinotectal tract 3-Tectospinal tract
3-Spinoreticular tract 4-Reticulospinal tracts
4-Spinoolivary tract 5-Olivospinal tract
Prof.Adel Kamel 7

SHORT TRACTS
Short tracts (fig.1-2) are association tracts connecting various levels of the spinal cord
together.They are related to spinal reflex mechanism. Impulses entering the spinal cord at
any segment may travel along these short tracts to higher or lower levels before connecting
directly or through internuncial neurons with the anterior horn cells. The short tracts are:
1-Fasciculus proprious (fundamental column).
2-Septomarginal tract.
3-Comma tract (fasciculus interfascicularis).
4-Dorsolateral tract (zone of Lissauer).

Spinal reflexes
A reflex is an involuntary response to a sensory stimulus. A spinal reflex varies
quantitatively as a result of supraspinal and intersegmental influences. Pathways mediated
by a reflex consist of afferent and efferent neurons.
Stretch reflex
If a muscle is stretched it responds by contraction. Stretch reflexes are important in control
of muscle tone (degree of resistance to passive movements) and body posture

Flexor reflex
Painful or thermal stimulation excites receptors and impulses are transmitted to motor
neurons through interneurons. This results in ipsilateral excitation of flexors and inhibition
of extensors. Also, there is contralateral inhibition of flexors and excitation of extensors.
This is termed crossed extensor reflex. Descending pathways normally inhibit flexor
reflexes unless this reflex is painful.

Clinical notes
-Spasticity is abnormal increase in muscle tone seen in pyramidal tract lesion. If stretch is
applied to a spastic muscle it suddenly gives way (klasp-knife).
-Rigidity is abnormal increase in muscle tone. Rigidity has phasic quality that is called cog-
wheel rigidity seen in Parkinson's disease.
-Flaccidity is absence of muscle tone as seen in lower motor neuron lesion.
Prof.Adel Kamel 8

LONG ASCENDING TRACTS

General consideration

The conscious sensory information reaches the cerebral cortex where sensations are felt.
Painful and thermal sensations ascend in the lateral spinothalamic tract; light touch and
pressure ascend in the anterior spinothalamic tract. Gracile and cuneate tracts carry
discriminative touch, position, movement and vibratory sensations. Unconscious
information does not reach the cerebral cortex; it carries impulses that are not felt.
Information from muscles, joints, skin and subcutaneous tissue reaches the cerebellum by
way of the anterior and posterior spinocerebellar tracts. Pain, thermal and tactile information
is passed to the superior colliculus of the midbrain through the spinotectal tract for the
purpose of activating spinovisual reflexes. The spinoreticular tract provides a pathway from
muscles, joints and the skin to the reticular formation. The spinoolivary tract provides an
indirect pathway for further information to reach the cerebellum.

Lateral spinothalamic tract (fig.1- 3)

Function: carries pain and temperature sensation from the body.

Receptors: free nerve endings (pain), Ruffini organ (heat) and Krause bulb (cold) senses.

First order neuron: are small cells in the posterior root ganglion. Axons of PRG enter the
spinal cord in the lateral division of the posterior root to the dorsolateral tract of Lissauer.

Second order neuron: includes the substantia gelatinosa of Rolando. Axons cross to the
opposite side in the anterior gray and white commissures to ascend in the contralateral white
column as the lateral spinothalamic tract. Fibers cross the spinal cord obliquely and reach
their tract 3 or 4 segments higher than their posterior root. As the tract ascends through the
spinal cord, new fibers are added to its anteromedial aspect of, so that in the upper cervical
segments the cervical portion is anteromedial while the sacral portion is posterolateral.
Fibers carrying pain are anterior to those conducting temperature. The lat.spinothalamic T
ascends in the medulla and is joined by the ant.spinothalamic T. to form the spinal lemniscus
which continues to ascend in the pons and midbrain to end in the thalamus.

Third order neuron: is the ventral posterolateral nucleus of the thalamus (PLVNT). Here
crude pain and temperature are appreciated. Axons of the third order neuron pass through
the posterior limb of the internal capsule to end in the sensory area (area 3,1,2).
Prof.Adel Kamel 9

Anterior (ventral) spinothalamic tract (fig.1-4)


Function: carries light (crude) touch and pressure sensation.
Receptors: Merkel's disc and Meissner's corpuscle.
First order neuron: is PRG cells, axons of which proceed to tip of posterior gray matter
where they divide into ascending and descending branches contributing to Lissauer's tract.

Second order neuron: is the substantia gelatinosa. Axons of the second order cross very
obliquely to the opposite side in the anterior gray and white commissures and ascend in the
contralateral anterior white column as the anterior (ventral) spinothalamic tract. As the tract
ascends in the spinal cord new fibers are added to its medial aspect. In the medulla the tract
accompanies the lat.spinothalamic T and the spinotectal T, all of which form the spinal
lemniscus which ascends in the pons and midbrain.

Third order neuron is PLVNT. Axons of the third order neuron pass through posterior
limb of the internal capsule and the corona radiata to end in the primary sensory area.

Clinical notes
Destruction of ant.spinothalamicT. produces contralateral deficit of light touch and pressure.
Bilateral destruction causes complete loss of itching, tickling and pleasant libidinous feeling.
-Lesion of one lateral spinothalamic tract produces analgesia and thermoanesthesia of the
contralateral side of the body below the level of the lesion. Visceral pain is preserved
because it is more diffuse (polysynaptic) and is bilaterally represented.
-Substance P is the neurotransmitter at the synapse with substantia gelatinisa.
-Types of pain are sharp, dull (acking), pricking, hyperalgesia (exaggerated feeling of pain),
immediate and long-lasting pain. Sharp pain terminates in the PLVNT and then sensory
cortex. Burning pain terminates in the reticular formation and then activates the entire CNS.
-Control of pain
a)Analgesia system secretes morphia-like substances (endorphins and encephalins) which
inhibit release of substance P. This system is found in the periaqueductal and periventricular
gray matter.
b)Inhibition of pain by non-painful touch stimuli, this interrupts (close the gate) for pain.
c)Surgical relief of pain is by stereotactic lesions. Rhizotomy is damage of posterior roots of
spinal nerves. This will deprive the patient from other senses. Cordotomy or tractotomy is
interruption of lateral spinothalamic tract by inserting a knife into anterolateral quadrant
of the spinal cord just ventral to denticulate ligament.
Prof.Adel Kamel 10

Fig.1-3. Pain and temperature pathway.


Prof.Adel Kamel 11

Fig.1-4. Crude touch pathway.


Prof.Adel Kamel 12

Gracile and cuneate tracts (fig.1-5)

Function: Gracile and cuneate tracts carry discriminative touch, vibratory and conscious
muscle and joint senses (kinesthesis). Fasciculus gracilis contains fibers from the lower part
of the body while fasciculus cuneatus contains fibers from the upper trunk, neck and upper
extremities.

Receptors: Pacinian corpuscle (vibration), Meissner's corpuscle (two point discrimination),


muscle spindle (muscle movements) and Golgi tendon organ (position of the body).

First order neuron: Large cells in the posterior root ganglion. Medial division of the
posterior root enters the posterior white column of the same side. Here fibers divide into
long ascending and short descending tracts. The descending branches give collaterals that
synapse with cells of the posterior gray horn, inter-nuncial neurons and anterior horn cells.
These short descending fibers are involved with inter-segmental reflexes. The long
ascending fibers travel upward in gracile and cuneate tracts.

Second order neuron is the gracile and cuneate nuclei of the medulla oblongata. Axons of
the second order neurons, called the internal arcuate fibers, sweep anteromedially around the
central gray matter and cross the median plane, decussating with the corresponding fibers of
the opposite side in the sensory decussation. Fibers ascend as a single compact bundle,
medial lemniscus, through the brain stem to terminate in the third order neuron.

Third order neuron is the PLVNT. Axons of the PLVNT pass through the posterior limb
of the internal capsule and corona radiata to end in the sensory area where fine touch,
vibratory sense and the position of the different parts of the body are appreciated.

-A number of fibers from nucleus cuneatus enter the cerebellum through the inferior
cerebellar peduncle of the same side. The pathway is referred to as the cuneocerebellar tract
and the fibers are known as the posterior external arcuate fibers. These fibers convey
information of muscle and joint senses of the upper limb to the cerebellum

- Destruction of the posterior white column cuts off the supply of information from the
muscles and joints to consciousness; thus there is ipsilateral loss of position, movement and
vibration senses. The patient has impaired muscular control and his movements are ataxic
(sensory ataxia). There is ipsilateral loss of tactile localization and two point discrimination
below the level of the lesion.
Prof.Adel Kamel 13

Fig.1-5. Proprioceptive pathway.


Prof.Adel Kamel 14

Posterior (uncrossed) spinocerebellar tract (fig.1- 6)

Function: The posterior (dorsal) spinocerebellar tract conveys proprioceptive information


concerning movements of muscles of the lower limb and trunk to the cerebellum. The
cerebellum uses this information to coordinate limb movements and maintain posture.

Receptors: muscle spindle and tendon spindle.

First order neuron is PRG. Axons of the PRG enter the posterior gray matter as collaterals
from the posterior white column, and terminate by synapsing with:

Second order neuron is the nucleus dorsalis or Clarke's nucleus. This nucleus is present
at the base of the posterior gray matter (lamina VII) and extends from C8 to L3 spinal cord
segments. Axons of Clarke's nucleus enter the posterolateral part of the lateral white matter
of the same side and ascend as the posterior spinocerebellar tract to the medulla oblongata.
Here the tract joins inferior cerebellar peduncle and terminates in the cerebellar cortex.

Anterior (crossed) spinocerebellar tract (fig.1-6)

Function: The anterior (ventral) spinocerebellar tract conveys proprioceptive information to


the cerebellum concerning muscle position and movements of the trunk and lower limbs. It
may transmit exteroceptive information to the cerebellum.

Receptors: muscle spindle, tendon spindle and probably exteroreceptors.

First order neuron: Cells of PRG. Axons of PRG enter the gray matter as collaterals from
the posterior white column to terminate in the second order neuron which is the spinal
border nucleus. These are widely distributed cells at the base of the anterior gray column
in lamina VII. This nucleus extends from L1 to S2 spinal cord segment. Axons of the second
order neuron cross to the opposite side and ascend in the contralateral white column as the
anterior spinocerebellar tract. Fibers ascend in the brain stem and enter the cerebellum
through the superior cerebellar peduncle and terminate in the cerebellar cortex. It is believed
that these fibers cross back into the cerebellum.

-Lesion of the posterior spinocerebellar tract results in ipsilateral leg dystaxia. Lesion of
the ventral spinocerebellar tract results in contralateral leg dystaxia.
Prof.Adel Kamel 15

Fig.1-6. Spinocerebellar tracts.

Spinotectal tract

Function: The spinotectal tract provides information for the spinovisual reflexes and brings
about movements of the head and eyes towards the source of stimulation.

First order neuron is the posterior root ganglion.

Axons from the posterior root ganglion enter the gray matter of the spinal cord to synapse
with the second order neuron which is the main sensory nucleus (nucleus proprious)
situated deeply in the posterior gray matter. Axons of the second order neuron cross to the
opposite anterolateral white column and then ascend through the brain stem to terminate in
the superior colliculus of the midbrain.
Prof.Adel Kamel 16

Spinoolivary tract

Function: The spinoolivary tract conveys information to the cerebellum from proprioceptive
and cutaneous receptors.

First order neuron is the PRG. Axons enter the gray matter as collaterals from the
posterior white column, to synapse with the second order neuron which is the nucleus
proprious. Axons then cross the median plane and ascend as the spinoolivary tract found at
the junction of the anterior and lateral white columns to synapse with the third order
neuron which is the dorsal and medial accessory olivary nuclei. Axons cross back the
midline to enter inferior cerebellar peduncle to terminate in the cerebellar cortex.

Spinoreticular tract

Function: The spinoreticular tract provides afferent pathways to the reticular formation
which plays a significant role in maintenance of the state of awareness. Also, the lateral
reticular nucleus of the medulla projects to specific portions of the cerebellum concerned
with transmission of exteroceptive impulses to the cerebellum.

First order neuron is PRG. Axons enter the spinal cord and terminate on unknown second
order neuron in the posterior horn. Axons from theses cells ascend bilaterally in the lateral
white column mixed with the lateral spinothalamic tract. Fibers terminate synapsing with
neurons of the reticular formation in the medulla, pons and midbrain.

Clinical notes
1-Tabes dorsalis is caused by syphilis. The disease causes selective destruction of nerve
fibers at point of entrance of the posterior root into the spinal cord, especially in the lower
thoracic and lumbosacral regions. It may manifest by severe pain or parathesia in the lower
limbs, loss of sensation in the trunk and lower limbs, loss of awareness that the bladder is
full, loss of appreciation of posture and movement of limbs, ataxia of the lower limbs, and
loss of tendon reflexes (due to degeneration of the afferent component of the reflex arc) .
2-Syringomyelia is central gliosis and cavitation of the spinal cord as a result of abnormal
formation of the central canal. It results in interruption of the lateral and anterior
spinothalamic tracts as they cross in the anterior white and gray commissures. The patient
has segmental bilateral loss of pain and thermal senses and some impairment of touch. As
the cavitation expands, other tracts and nerve cells become involved such as anterior horn
cells and corticospinal tracts.
Prof.Adel Kamel 17

LONG DESCENDING TRACTS

Pyramidal tract (fig.1-7)

Function: The pyramidal tract is the main motor tract. It is the pathway concerned with
voluntary, discrete, fractionated and skilled movements.

Origin: One-third of fibers of the tract arises from the pyramidal cells (Betz Cells) in the
primary motor area (area 4), one-third from the secondary motor area (premotor area 6), and
one-third from other areas (e.g. sensory area 3,1&2). The body is represented in a precise
somatotopic fashion in an inverted manner, with the region controlling the face situated
inferiorly and that controlling the lower limb situated superiorly a. The area of the cortex
devoted to a particular body part is proportional to the degree of precision with which
movements can be executed. Therefore, the tongue, face and fingers are represented by
relatively large regions. The pyramidal tract comprises the Corticobulbar (Corticonuclear)
tract and Corticospinal tracts.

Corticobulbar (Corticonuclear) tract

The corticobulbar tract descends in the corona radiata, genu of the internal capsule, crus
cerebri of the midbrain just medial to the corticospinal tract and ends by synapsing with
cranial nerve nuclei of the same and opposite side (bilateral innervation).

Bilateral projections are evident to nuclei innervating muscles which can not contract
voluntarily on one side. These include muscles of palate, larynx, pharynx, extraocular
muscles, muscles of mastication and upper face. The lower facial and genioglossus muscles
receive corticonuclear innervations mostly from opposite side. Most fibers regarded as
corticobulbar are distributed to neurons in the reticular formation which in turn relay
impulses to the motor cranial nerve nuclei. However, there are direct corticobulbar fibers to
trigeminal, facial and hypoglossal nuclei. Some corticobulbar fibers end in sensory nuclei of
brain stem (gracile, cuneate, solitary, and trigeminal). These fibers inhibit or stimulate
sensory transmission allowing selective attention or selective inattention to various stimuli.
Prof.Adel Kamel 18

Corticospinal tract

The descending pyramidal fibers converge in the corona radiata and then pass in the
posterior limb of the internal capsule. The tract then descends through the middle three fifths
of crus cerebri of the midbrain. On entering the pons the corticospinal tract is broken into
many bundles (dispersed pyramidal tract) by the transverse pontine fibers. In the medulla,
the bundles become grouped together to form the pyramid. At the lower level of the
medulla most of the fibers cross the midline at motor decussation and enter the dorsal
quadrant of the lateral white column of the spinal cord to form the lateral corticospinal
tract. The remaining uncrossed fibers (10-25%) descend in the anterior white column of the
spinal cord as the anterior (ventral) corticospinal tract. These fibers eventually cross the
midline and terminate in the anterior gray column of the spinal cord segments of the cervical
and upper thoracic regions. Fibers of the lateral corticospinal tract are about one million and
the tract is the longest continuous tract in the CNS; it descends the whole length of the spinal
cord and it gradually diminishes in size as it descends. Its fibers end in the anterior gray
matter of all the spinal cord segments; 55% end in the cervical region, 20% in the thoracic,
and 25% in the lumbosacral region. This would suggest that pyramidal control over the
upper limbs is much greater than the lower limbs. Most of the lateral corticospinal fibers
(90%) enter the gray matter and synapse with internuncial neurons in laminae IV-VII
before synapsing with AHCs. Only the largest corticospinal fibers (about 10% of the tract
fibers) synapse directly with the motor neurons (anterior horn cells) of lamina IX. Fibers of
the large Betz cells (about 3% of the tract), large myelinated fibers of 20 micron diameter,
are concerned with the finer isolated movements of the distal parts of extremities while
thinner fibers are related to grosser movements and tonic control.
Prof.Adel Kamel 19

Fig.1-7. Pyramidal tract.


Prof.Adel Kamel 20

Clinical notes
1-Unilateral lesion interrupting the corticobulbar tract (upper motor neuron lesion) produces
mild form of paresis because of the bilateral cortical control of the motor cranial nerve nu-
clei. There is deviation of the tongue to the opposite side of the lesion and marked weakness
of the lower facial muscles of the opposite side of the lesion (angle of the mouth deviates to
the same side of the lesion).
2-Bilateral lesion involving corticobulbar fibers produces a syndrome known as
pseudobulbar paralysis. This syndrome is characterized by paralysis of muscles of
swallowing, chewing, speaking and breathing.
3-Lesion of the corticospinal tract rostral to motor decussation produces contralateral loss of
voluntary movements that is more marked in the distal parts of the extremities. Lesion of the
corticospinal tract caudal to the motor decussation results in ipsilateral paralysis below the
level of the lesion. It is also a spastic upper motor neuron lesion. In spastic hemiplegia there
is an increase in the deep reflexes (hyperreflexia). Babinski sign is positive.
4-Types of paralysis; hemiplegia is paralysis of one side of the body, monoplegia is
paralysis of one limb only, diplegia is paralysis of two corresponding limbs (arms or legs),
quadriplegia is paralysis of all 4 limbs, paraplegia is paralysis of the two lower limbs.
Alternating hemiplegia is contralateral paralysis of the body due to upper motor neuron
lesion of the pyramidal tract in the brain stem, associated with lower motor ipsilateral
paralysis of one or more of the motor cranial nerves.

Extrapyramidal tracts

The term extrapyramidal tracts refer to all tracts other than the corticospinal tract; the
descending fibers which are relayed in subcortical nuclei before reaching the spinal cord.

Reticulospinal tract

Function. The reticulospinal tracts facilitate or inhibit the activity of the motor neurons; by
this it influences the voluntary movements and reflex activities. The tracts include also
descending autonomic fibers, thus providing a pathway by which the hypothalamus can
control the sympathetic and sacral parasympathetic outflows.

Origin: Group of scattered neurons in the brain stem known as the reticular formation.

Descent: Reticulospinal tracts are lateral crossed (medullary) and anterior uncrossed
(pontine) tracts. Both sets enter the gray matter and end by synapsing (directly or through
internuncial neurons) with the end neurons which are the anterior or lateral horn cells of the
gray matter of the spinal cord.
Prof.Adel Kamel 21

Olivospinal tract

Function: unknown, it may influence the activity of motor neurons. The olivospinal tract
arises from inferior olivary nucleus, descends in the lateral white column of the spinal cord
and end in the anterior gray mater. Existence of the olivospinal tract is now questionable.

Rubrospinal tract

Function: The rubrospinal tract facilitates the activity of the flexor muscles.

Origin: Red nucleus of the midbrain. Axons of the cells in this nucleus cross immediately
the midline at the ventral tegmental decussation of the midbrain to form the rubrospinal
tract.

Descent: The rubrospinal tract descends through the pons and medulla to enter the lateral
white column of the spinal cord intermingled with fibers of the lateral corticospinal tract.
The fibers synapse with internuncial neurons in the anterior gray matter of the spinal cord
and then through these neurons the tract can affect the anterior horn cells. The red nucleus
receives fibers from the cerebral cortex and the cerebellum. This is an indirect pathway by
which the cerebral cortex and the cerebellum can influence the activity of the alpha and
gamma neurons of the spinal cord. Conclusions that the rubrospinal tract in man is rudim-
entary are based on the fact that cells of the human red nucleus are small and also the
rubrospinal tract is not demonstrated below the thoracic spinal segments.

Vestibulospinal tract

Function: The vestibulospinal tract facilitates the activity of the extensor muscles in
association with maintenance of balance.

Origin: Neurons of the lateral vestibular nucleus.

Descent: The tract descends uncrossed in the anterior white column of the spinal cord and
synapses with internuncial neurons, which influences the anterior horn cells. The cerebellum
and inner ear, by means of this tract, maintain balance and posture.
Prof.Adel Kamel 22

Tectospinal tract

Function. The tectospinal tract is concerned with reflex postural movement of the upper
body in response to visual and to auditory stimuli.

Origin: Superior colliculus. Fibers cross the midline in the posterior tegmental decussation.

Descent: The tectospinal tract descends through the brain stem close to the medial
longitudinal bundle and descends through the anterior white column of the spinal cord close
to the anteromedian fissure. Majority of fibers terminate in the upper cervical segments by
synapsing with internuncial neurons which in turn influence the anterior horn cells.

Fig.1-8.Transverse section of the cervical spinal cord showing on the left gray horns,
ascending and descending pathways. Deficits resulting from individual lesions of these
pathways are shown on the right.
Prof.Adel Kamel 23

DISORDERS OF THE SPINAL CORD (Fig.1-8)

A) Motorneuron diseases
1-Diseases of upper motor neurons
-Damage to cortical neurons or corticospinal tracts. It manifests with clasp-knife
spasticity, hyperreflexia and positive Babinski sign.

2-Diseases of lower motor neurons


- Poliomyelitis is an acute viral infection with a predilection to anterior horn cells causing
selective atrophy of relevant muscles.

B) Sensory neuron diseases


-Tabes dorsalis (syphilis): selectively affect dorsal columns. Patient loses proprioception
but retains pain.

C) Diseases with combined degeneration of dorsal and lateral columns


Subacute combined degeneration (posterolateral sclerosis) is caused by vitamin B12
deficiency. Patient has signs of UMNL and dorsal column sensory loss.

D) Hemisection of the spinal cord (Brown-Sequard syndrome)


Hemisection of the spinal cord may be caused by bullet or stab wounds, or by an expanding
tumor. Incomplete hemisection is more common than complete hemisection (fig.1-9).
Motor manifestations
Damage of anterior horn cells produces ipsilateral lower motor neuron (LMNL) paralysis at
level of segment of lesion with muscular atrophy. Damage of corticospinal tract produces
ipsilateral (UMNL) spastic paralysis below level of the lesion.
Sensory manifestations
Damage of gracile and cuneate tracts produces ipsilateral loss of proprioception below the
level of the lesion. Damage of the lateral spinothalamic tract produces contralateral loss of
pain and temperature senses two segments below the level of the lesion and further distal.
At the level of the lesion there is hypotonic paralysis and a band of total loss of all
sensations. This is due to destruction of the posterior root and its entrance into the cord.
Autonomic manifestations
Ipsilateral Horner's syndrome is due to damage of the descending sympathetic pathway, and
is characterized by ptosis (drooping of eye lid), miosis (papillary constriction) and
anhydrosis (dryness of skin).
Prof.Adel Kamel 24

Fig.1-9. Hemisection of the spinal cord (Brown sequard syndrome).


E) Complete transection of the spinal cord
This lesion may be caused by fracture dislocation of the vertebral column, by bullet or stab
wound or by expanding tumor. After a period of spinal shock, the following clinical
manifestations are observed:
1-Bilateral lower motor neuron lesion in the segment of the lesion.
2-Bilateral spastic paralysis below the level of the lesion.
3-Bilateral positive Babinski sign.
4-Bilateral loss of superficial reflexes.
5-Bilateral loss of all sensations below the level of the lesion
6-Bladder and bowel functions are not under voluntary control.

F) Conus medullaris syndrome


Conus medullaris syndrome involves segments S3-Co and is usually due to intramedullary
tumor, hemorrhage, herniated disc or trauma. It manifests by:
1-Urinary and fecal incontinence due to damage of sacral parasympathetic outflow.
2-Saddle anesthesia (peri anogenital sensory loss) due to dermatomes S3-Co loss.
3-Absence of motor deficits of lower limbs.

G) Cauda equine syndrome


It involves spinal roots L3-Co. Its manifestations are usually in one side.
1-Parathesia and radicular pain.
2-Motor disability of the lower limb especially the thigh.
3-May disturb bladder and rectum functions.
Prof.Adel Kamel 25

STUDY QUESTIONS ON CHAPTER 1

Choose ONLY one correct answer

1-A tract whose lesion results in 6-The Corticospinal tract doesn't receive
ipsilateral leg dystaxia is contribution from
a)Posterior spinocerebellar. a)Prefrontal cortex.
b)Anterior spinocerebellar. b)Premotor cortex.
c)Cuneate. c)Paracentral lobule.
d)Anterior spinothalamic. d)Primary sensory area.

2-Second order neuron of pain is 7-False about conus medullaris syndrome


a)Nucleus proprius. a)Impotence.
b)Substantia gelatinosa. b)Urinary incontinence.
c)Intermediomedial nucleus. c)Perianogenital sensory loss.
d)Commissural nucleus. d)Involves segments L4-S2.

3-A sign of lower motor neuron lesion 8-A patient who failed to recognize a coin in
a)Spasticity. his hand would have a lesion in
b)Hyperreflexia. a)Anterior spinocerebellar tract.
c)Positive Babinski. b)Cuneate tract.
d)Muscle wasting. c)Anterior spinothalamic tract.
d)Fasciculus proprius.

4-A tract activates antigravity muscles 9-Correct source for the dorsal column is
a)Rubrospinal . a)Sympathetic ganglion.
b)Tectospinal. b)Dorsal root ganglion.
c)Vestibulospinal. c)Substantia gelatinosa.
d)Olivospinal. d)Nucleus proprius.

5-False about stretch reflex


a)It includes muscle spindle. 10-Limb muscles are supplies by neurons
b)Afferent neurons are in dorsal horn. a)In posterior horn.
c)It includes anterior horn cells. b)In Lateral horn.
d)It is monosynaptic. c)In Anterior horn.
d)In dorsal root ganglion.
Prof.Adel Kamel 26

CHAPTER 2

BRAIN STEM

Objectives
-Describe external features of the brain stem
-Identification of the tracts and nuclei in the brain stem.
-Orientation of the cranial nerve nuclei in the brain stem.
-Explanation of the anatomical basis of lesions at different levels of the brain stem.
-Highlight the connections and significance of the reticular formation.

MEDULLA OBLONGA
Gross features (fig.2-1)
The medulla oblongata is conical (bulb) in shape broad superiorly and narrow inferiorly. It
connects the pons with the spinal cord. On each side of its anteromedian fissure there is a
swelling named the pyramid. The pyramid tapers inferiorly where majority of corticospinal
fibers cross to the opposite side. Posterolateral to the pyramids are the olives which are oval
elevations produced by the inferior olivary nuclei. Rootlets of the hypoglossal nerve emerge
between the pyramid and olive on each side. Posterior to the olives are the inferior cerebellar
peduncles. Rootlets of the glossopharyngeal, vagus and accessory nerves emerge in the
groove between the olives and inferior cerebellar peduncles.

The posterior surface of the rostral medulla forms the floor of the 4th ventricle. The posterior
surface of the caudal medulla possesses a posterior median sulcus on each side of which
there is an elongated swelling, the gracile tubercle produced by gracile nucleus. Lateral to
the gracile tubercle is a cuneate tubercle produced by cuneate nucleus.
Prof.Adel Kamel 27

Fig.2-1A.Ventral view of the brain stem and cranial nerves.

Fig.2-1B.Dorsal view of the brain stem and cranial nerves.


Prof.Adel Kamel 28

Internal structures of the medulla

Internal features of medulla are illustrated in transverse sections (fig.2-2) through 3 levels:

1-Caudal level, level of motor decussation.


2-Middle level, level of sensory decussation.
3-Superior level, level of open medulla.
Caudal medulla (Level of motor decussation)
At transition from the spinal cord to the medulla, the pattern of gray and white matter
undergoes considerable rearrangement. The ventral horn becomes much attenuated. The
dorsal horn is replaced by the spinal trigeminal nucleus. Superficial to this nucleus is the
spinal tract of the trigeminal nerve descending to the spinal trigeminal nucleus. In the ventral
aspect majority of pyramidal tract fibers decussate (motor decussation) and pass laterally,
dorsally and caudally to form the lateral corticospinal tract.

Middle medulla (Level of sensory decussation)

Gracile and Cuneate nuclei appear beneath their respective tracts. Axons of gracile and
cuneate nuclei course medially and ventrally as internal arcuate fibers which decussate at
this level forming sensory decussation. The decussated fibers turn rostrally forming medial
lemniscus (on each side) which ascends in the brain stem to end in the thalamus. Posterior
external arcuate fibers originate from accessory cuneate nucleus (positioned lateral to
cuneate nucleus) to reach the cerebellum through the inferior cerebellar peduncle.

Rostral (open) medulla

Inferior olivary nucleus is shown within prominence of the olive. It is present dorsolateral to
the pyramid. Lateral to the medial lemniscus is the spinothalamic tract. The dorsal surface of
open medulla forms part of the floor of the 4th ventricle. Immediately deep to this floor lie a
number of cranial nerve nuclei (hypoglossal nucleus, dorsal motor nucleus of the vagus
nerve, nucleus ambiguous, and inferior and medial vestibular nuclei). Ventromedial to the
hypoglossal nucleus close to the midline is the medial longitudinal bundle which links
vestibular with ocular nuclei. Dorsolateral prominence of open medulla is the inferior
cerebellar peduncle.
Prof.Adel Kamel 29

Fig.2-2. Transverse sections of the medulla.

A.Caudal level, Pyramidal (motor) decussation.


B.Middle level, Sensory decussation.
C.Rostral level (open) medulla.
Prof.Adel Kamel 30

Functional significance of the medulla oblongata


1-It contains several cranial nerve nuclei.
2-It offers passage for ascending and descending tracts.
3-It contains the medullary reticular formation.
4-It contains vital centers; the vomiting, respiratory and cardiovascular centers.
Blood supply of the medulla
Vertebral, anterior and posterior spinal, posterior inferior cerebellar and basilar arteries.
Clinical notes
1- Tumors of the posterior cranial fossa produce increased intra-cranial pressure. There is a
tendency to downward herniation of the medulla and cerebellar tonsils through the foramen
magnum.This will produce headache, neck stiffness and paralysis of the lower 4 cranial
nerves owing to traction. In this condition lumbar puncture is contraindicated.
2- Lateral medullary syndrome (of Wallenberg) results from thrombosis of the posterior
inferior cerebellar artery. It manifest by: a) Ipsilateral paralysis of palatal and laryngeal
muscles with dysphagia and dysarthria (affection of the nucleus ambiguous), b) Ipsilateral
thermoanesthesia of the face (lesion of spinal nucleus of trigeminal nerve), c) Vertigo,
nausea, vomiting and nystagmus (affection of vestibular nuclei), d) Ipsilateral Horner's
syndrome (affection of the descending sympathetic fibers), e) Gait and limb ataxia (lesion of
inferior cerebellar peduncle), and f) Contralateral loss of pain and temperature of the body
(lesion of the spinal lemniscus).Fig.2-3B.
3-Medial medullary syndrome is due to thrombosis of the medullary branches of vertebral
artery. It manifests by; a) Contralateral hemiplegia (pyramidal tract affection), b)
Contralateral impaired sensation of position and movement (lesion of the medial lemniscus),
and c) Ipsilateral paralysis of the tongue with deviation of its tip to the paralysed side
(affection of the hypoglossal nerve). Contralateral hemiplegia associated with lower motor
neuron lesion of a cranial nerve is called alternating hemiplegica (Fig.2-3A).

Fig.2-3. Medullary syndromes.


Prof.Adel Kamel 31

PONS

Gross features (fig.2-1)

The pons (bridge) connects the medulla to the midbrain. It is located ventral to the
cerebellum where it bridges to connect the two cerebellar hemispheres. The anterior surface
shows many transverse pontine fibers that converge on each side to form the middle
cerebellar peduncle. The trigeminal nerve fibers emerge from the anterolateral surface
between the pons and its peduncle. In the groove between the pons and the medulla emerge
the abducent, the facial and the vestibulocochlear nerves.

The posterior surface of the pons forms the upper half of the floor of the 4th ventricle. It is
divided into two halves by a median sulcus. Lateral to this sulcus is the facial colliculus
produced by fibers of the facial nerve winding around nucleus of the abducent nerve. Lateral
to the facial colliculus is the vestibular area produced by underlying vestibular nuclei.

Internal structures of the pons (fig.2-4) are studied in 3 levels:

1-Inferior level, level of facial colliculus.


2-Middle level, level of trigeminal nuclei.
3-Superior level, level of 4 lemnisci.

For purposes of description the pons is divided into a posterior part, the tegmentum, and an
anterior part, the basis pontis. Basis pontis is marked by transversely oriented
pontocerebellar fibers originating from scattered pontine nuclei and pass to the contralateral
cerebellar hemisphere through the middle cerebellar peduncle. Corticospinal fibers appear
dispersed between fascicles of transverse pontine fibers. Tegmentum pontis contains
reticular formation, lemnisci and cranial nerve nuclei.
Prof.Adel Kamel 32

Fig.2-4.Transverse section of the pons; A.Level of facial colliculus, B.Level of


trigeminal nuclei, C.Level of four lemnisci.
Prof.Adel Kamel 33

Caudal Pons (level of facial colliculus)

The trapezoid body is located ventral to ascending lemnsci but dorsal to basis pontis.
Beneath the floor of the 4th ventricle lie a number of cranial nerve nuclei. Abducent nucleus
is medially located and close to the surface. Facial motor nucleus is located ventrolateral to
abducent nucleus. Fibers of facial nerve wind around abducent nucleus before exiting from
the ventral aspect of the pons. This creates the facial colliculus seen in the floor of the 4th
ventricle. Laterally in the floor of the 4th ventricle is the vestibular area deep to which are the
superior and lateral vestibular nuclei. The cochlear nuclei are located more laterally
posterior and anterior to the inferior cerebellar peduncle. Lateral lemniscus of the auditory
pathway is seen in the tegmentum of the caudal pons.

Mid Pons (level of trigeminal nuclei)

The four trigeminal nuclei reach their maximum extent at this level adjacent to the origin of
the trigeminal nerve which exits from the ventral aspect between the pons and its peduncle.

Rostral Pons (level of four lemnisci)

The superior cerebellar peduncles form the lateral walls of the 4th ventricle. The thin
superior medullary velum spans between the superior cerebellar peduncles to form roof of
the 4th ventricle. The 4 lemnisci are organized in order as medial, spinal, trigeminal and
lateral forming crescent of lemnsci.

Functional significances of the pons


1-Contains cranial nerve nuclei: V,VI VII, and VIII.
2-Pathway for ascending and descending tracts.
3-Pathway for hearing.
4-Contains pontine reticular formation and superior salivatory nucleus.
5-Contains part of respiratory centers.

Blood supply of the pons


The pons receives arterial blood supply from pontine branches of the basilar artery, branches
from anterior inferior cerebellar and superior cerebellar arteries.
Prof.Adel Kamel 34

Clinical notes

Lesions of the pons can be vascular, neoplastic, traumatic or demyelinating. Manifestations


vary according to if the lesion is unilateral or bilateral, medial or lateral, located in the
caudal or rostral pons, and involving basis or tegmentum pontis. A unilateral lesion usually
manifests by contralateral hemiplegia (affection of pyramidal tract) and ipsilateral cranial
nerve dysfunction (trigeminal, abducent, facial or vestibulocochlear). A bilateral lesion may
destroy the vital centers for respiration and circulation leading to coma and death.

Fig.2-5. Vascular lesions of the pons (A) Medial inferior pontine syndrome. (B) Lateral
inferior pontine syndrome.

MIDBRAIN
Gross features (fig.2-1)
The midbrain connects the pons and cerebellum with the forebrain. It is traversered with the
cerebral aqueduct which connects the 3rd and 4th ventricles. On the ventral surface there is a
depressed area called the interpeduncular fossa bounded on each side by the crus cerebri.
Oculomotor nerve fibers emerge from a groove on the medial side of the crus cerebri. On the
dorsal aspect there are 4 colliculi (4 corpora quadrigemina); two superior and two inferior
colliculi. The superior colliculi are centers for visual reflexes. The inferior colliculi are
centers for auditory reflexes. In the midline below the colliculi the trochlear nerves emerge
and then wind around the lateral aspects of the midbrain to enter the cavernous sinuses. On
the lateral aspects of the midbrain the superior and inferior brachia ascend to the lateral and
medial geniculate bodies of the thalamus, respectively.
Prof.Adel Kamel 35

Internal structures (fig. 2-6)


The cerebral aqueduct divides the midbrain into posterior part, tectum, and anterior part,
the cerebral peduncle. The cerebral peduncle is divided by the substantia nigra into an
anterior part, crus cerebri, and a posterior part, the tegmentum. In the periaqueductal gray
matter is nucleus of locus ceruleus (noradrenergic nucleus). Moreover there is the raphe
nucleus (serotonergic neurons). The tegmentum is bounded by the crus cerebri (cerebral
peduncle) which consists of descending cortical efferents. Its middle half is composed of
corticobulbar (medially) and corticospinal (laterally) fibers. On each side of the pyramidal
tract are the corticopontine fibers. The most ventral part of the tegmentum is occupied by
the substantia nigra composed of pigmented neurons that synthesize dopamine. These
neurons project to the corpus striatum.

The inner structures of the midbrain are studied in two levels:


1- Caudal level, level of inferior colliculi.
2- Rostral level, level of superior colliculi.

Caudal Midbrain
The inferior colliculus is part of the hearing system. At this level the ascending auditory
fibers in the lateral lemniscus end in the inferior colliculus. Leminiscal crescent is obvious at
this level. Ventral to the periaqueductal gray matter lays the trochlear nucleus. Close to it is
the MLB beneath the inferior colliculus.The superior ceebellar peduncles decussate in the
midline of this level. Periaqueductal gray matter contains several nuclei:

a)Nucleus of locus ceruleus (noradrenergic/dopaminergic). This nucleus extends to the pons


and is concerned with cognitive mapping, affective behavior, thalamic and neocortical
activities.

b)Raphe nucleus (serotinergic).

c)Dorsal tegmental nucleus (secretes endorphins and enkephalins). Close to this area is the
central tegmental tract containing the reticulothalamic tract (reticular activating system).
Prof.Adel Kamel 36

Rostral Midbrain
The superior colliculus is part of the visual system. Its afferents are mainly corticotectal
fibers originating from the visual cortex and the frontal eye field. These inputs are concerned
with controlling movements of the eyes (following objects, accommodation and conjugate
eye movements). Efferents from the superior colliculi decussate in the dorsal tegmental
decussation and descend as tectospinal tract. A small number of fibers of the optic tract
terminate in the pretectal nucleus, just rostral to the superior colliculus. The pretectal
nucleus is connected to Edinger Westphal (parasympathetic) nucleus to control papillary
light reflexes. Ventral to the periaqueductal gray matter are the oculomotor nuclei. The
central portion of the tegmentum is occupied by the red nucleus. Efferents from the red
nucleus decussate in the ventral tegmental decussation and descend as rubrospinal tract.

Functional significances of the midbrain


1-It contains cranial nerve nuclei: III and IV.
2-The cerebral aqueduct passes through it.
3-It offers pathway for the corticofugal fibers and the lemnisci.
4-The substantia nigra is considered part of the basal ganglia.
5-It has the red nucleus which is one of the extrapyramidal system.
6-It contains midbrain reticular formation.

Blood supply of midbrain is derived from the posterior cerebral and anterior choroid
arteries.
Clinical Notes
1-Blockage of the cerebral aqueduct; congenital or acquired (tumor) leads to accumulation
of the CSF within the third and lateral ventricles. This will result in obstructive
hydrocephalus.
2-Medial midbrain (Weber's) syndrome is due to occlusion of a branch of the posterior
cerebral artery which supplies the midbrain. It manifests by necrosis of the oculomotor
nerve and the crus cerebri, resulting in alternating hemiplegia; contralateral hemiplegia with
ipsilateral oculomotor paralysis (ophthalmoplegia). The ipsilateral eye is deviated laterally
and downwards. There is drooping (ptosis) of the upper eye lid. The pupil is dilated and
fixed in the same side of the lesion (fig.2-7).
3-Paramedian midbrain (Benedikt's) syndrome is vascular lesion with necrosis of the medial
lemniscus and the red nucleus. It is manifested by contralateral loss of proprioception from
trunk and limbs, and involuntary movements of the opposite limbs. Involvement of
oculomotor roots results in ptosis, ophthalmoplegia and ipsilateral dilated and fixed pupil.
4-Dorsal midbrain (Parinaud's) syndrome is mostly due to pinealoma resulting in:
-Paralysis of upward and downward gaze, papillary disturbances and absence of
convergence. This is due to lesion of the superior colliculi and pretectal area.
Prof.Adel Kamel 37

Fig.2-6.Transverse sections of the midbrain. (A) level of inferior colliculus, (B) level of
superior colliculus.

Fig.2-7. Midbrain syndromes. (A)Dorsal midbrain (Parinaud) syndrome. (B)


Paramedian midbrain (Benedikt) syndrome. (C) Medial midbrain (Weber syndrome).
Prof.Adel Kamel 38

CRANIAL NERVE NUCLEI IN THE BRAIN STEM

General consideration

Some cranial nerves are entirely sensory (I, II & VIII), some are entirely motor (III, IV, VI,
XI & XII) the rest are mixed (V, VII, IX & X). The motor fibers of the cranial nerves are
axons of nerve cells situated within the brain, nuclei of origin. The sensory parts of a cranial
nerve are the axons of cells outside the brain. These nerve cells are situated in ganglia on the
nerve trunks or may be present in a sensory organ such as the ear. Central processes of these
cells enter the brain and synapse with nuclei of termination.
Spinal nerves possess general somatic sensory fibers, general visceral sensory fibers, general
somatic motor fibers, and general visceral motor fibers. Cranial nerves, in addition, possess
special somatic sensory fibers (vision and hearing), special visceral sensory fibers (smell and
taste), and special visceral motor fibers (to muscles derived from the branchial arches).
The distribution of the cranial nerve nuclei in the brain stem tends to follow linear alignment
of each group according to its function (fig.2-8):
1-General somatic efferent (GSE) supply muscles derived from myotomes. The nuclei are
arranged in a linear order in the brain stem near the midline and close to the floor of the 4th
ventricle. They include the motor nuclei of III, IV, VI & XII.
2-General visceral efferent (GVE) are parasympathetic outflows in the cranial nerves
including III, VII, IX & X. The nuclei of which are, respectively, the Edinger Westphal, the
superior salivatory nucleus, the inferior salivatory nucleus and the dorsal motor nucleus of
the vagus.
3-Special visceral efferent (SVE) nuclei give nerves which supply muscles of the branchial
arches; V, VII, IX, X & XI. Their nuclei are, respectively, motor nucleus of trigeminal,
motor nucleus of facial and nucleus ambiguous (for IX, X & XI).
4-Special visceral afferent (SVA) for smell and for taste sensations. Taste is transmitted by
VII, IX & X cranial nerves. The second order neuron in taste pathway is called the gustatory
nucleus which is the superolateral part of the nucleus solitarius. Olfactory nerve is classified
as SVA.
5-Special somatic afferent (SSA) present in the vestibulocochlear nerve concerned with
equilibrium and hearing. The cochlear nuclei are two; dorsal and ventral, and the vestibular
nuclei are four; medial, lateral, superior and inferior. Optic nerve is classified as SSA.
6-General somatic afferent (GSA) concerned with sensations from head region. The
deep nuclei are the trigeminal nerve nuclei which are the mesencephalic, sensory and spinal
nuclei. These nuclei are considered as the second order neuron in the sensory pathway; the
first order neuron is the trigeminal ganglion. Ttrigeminal sensory nuclei, also, receive
somatic afferents of the facial, glossopharyngeal and vagus nerves (sensation from the ear).
Prof.Adel Kamel 39

7-General visceral afferent (GVA) concerned with sensation from viscera, tongue,
pharynx and larynx. Nuclei of the second order neuron are situated in the middle part of the
nucleus solitarius which receives afferents from glossopharyngeal and vagus nerves.
Three of cranial nerves possess five axonal componenets; Facial nerve,
Glossopharyngeal nerve and Vagus nerve. These componenets are GVE, SVE, SVA,
GVA, and GSA.

Fig.2-8. Cell columns of the seven cranial nerve modalities in the brain stem.
Prof.Adel Kamel 40

DEEP ORIGIN (NUCLEI) OF CRANIAL NERVES

Fig.2-9. Brain stem showing location of the motor and sensory cranial nerve nuclei.
Prof.Adel Kamel 41

I- OLFACTORY NERVE: see olfactory pathway.

II- OPTIC NERVE: see visual pathway.

III- OCULOMOTOR NERVE NUCLEI

The nuclei of the oculomotor nerve are present ventral to the central gray matter of the
midbrain in a V-shaped manner and extend from the upper part of the trochlear nucleus to
the upper part of the superior colliculus. The nuclear complex is formed of:

1-Lateral group of large cells which can be divided into:


a) Dorsolateral nucleus to inferior rectus.
b) Intermediate nucleus to inferior oblique.
c) Ventromedial nucleus to medial rectus.
2-Central caudal group to levator palpebrae superioris.
3-Medial group to the superior rectus.
4-Edinger Westphal nucleus (GVE): dorsal to the main nuclei of the oculomotor nerve.
Axons of its cells are preganglionic parasympathetic which relay in the ciliary ganglion.
The postganglionic fibers supply the constrictor pupillae and ciliary muscles.

Connections
The oculomotor nuclei receive corticonuclear fibers from both pyramidal tracts. They
receive tectobulbar fibers from the superior colliculus which transmit to them information
from the visual cortex. They, also, receive fibers from the medial longitudinal bundle by
which they are connected to nuclei of the fourth, sixth and eighth cranial nerves. Edinger
Westphal nucleus receives corticonuclear fibers for accommodation reflex, and fibers from
the pretectal nucleus for the direct and consensual light reflexes.

IV- TROCHLEAR NUCLEUS

The trochlear nerve nucleus (GSE) lies in the floor of the cerebral aqueduct at the level of
the inferior colliculus of midbrain. It supplies the superior oblique muscle.

Connections
The trochlear nucleus is connected to both corticonuclear tracts (motor order), medial
longitudinal bundle (association with III, VI & VIII), and tectobulbar tract (information
from visual cortex.
Prof.Adel Kamel 42

V- TRIGEMINAL NUCLEI

A- Motor nucleus (SVE): an ovoid column of multipolar cells that lies medial to the main
sensory nucleus. It occupies the position of the special visceral efferent column in the pons.
It supplies muscles of mastication, tensor palati and tensor tympani.

Connections
-Both corticonuclear tracts (motor control).
-Sensory nuclei of trigeminal (reflex arc for propriocetive control of masticatory muscles).
-Reticular formation, red nucleus, tectum and medial longitudinal bundle (correlation of
salivation and mastication).

B- Sensory nuclei, include:


1) Main sensory nucleus; lies lateral to entering trigeminal fibers and extends from the
upper level of pons to the upper level of the spinal nucleus of trigeminal. It is concerned
with sensations of touch and pressure of the face. Fibers of the ophthalmic nerve terminate
anteriorly, fibers of the maxillary are intermediate and fibers of the mandibular are most
posterior.

2) Spinal nucleus; extends from the lower part of the main sensory nucleus in the pons
inferiorly through the whole length of the medulla and into the upper part of the cervical
spinal cord segments. It receives sensations of pain and temperature from the face. Fibers
from ophthalmic nerve terminate in the lower part of the nucleus, fibers from maxillary
nerve terminate in the middle and fibers from mandibular terminate in the upper part of the
nucleus.

3) Mesencephalic nucleus; unipolar nerve cells situated in the lateral part of the gray
matter around the cerebral aqueduct in the midbrain and extends inferiorly into the pons as
far as the main sensory nucleus. It receives propriocetive impulses from muscles of
mastication and from facial and extraocular muscles.
Prof.Adel Kamel 43

PATHWAY OF SENSATIONS FROM THE FACE

Sensations from the face (fig.2-10) are carried by the trigeminal nerve and the first order
neuron is the cells of the trigeminal ganglion. Fibers then enter the pons where the second
order neuron is present which is the trigeminal nuclei. The sensory trigeminal nuclei are; a)
spinal nucleus which is concerned with pain and temperature sensation of the face, b) the
main sensory nucleus which is concerned with touch and pressure, and c) the
mesencephalic nucleus which is concerned with proprioception of the face. Fibers from
the trigeminal nuclei cross to the opposite side and ascend as the ventral
trigeminothalamic tract (trigeminal lemniscus). The trigeminal lemniscus terminates in
the third order neuron which is the PMVNT. Posterior medial ventral nucleus of the
thalamus (PMVNT) projects to the lower part of the primary sensory area (area 3, 1, 2).
Fibers reach the mesencephalic nucleus without relay in trigeminal ganglion. After relay in
mesencephalic nucleus fibers ascend in the same side as the dorsal trigeminothalamic
tract to reach the PMVNT.

Fig.2-10.Trigeminal nuclei and pathway of sensations from the face.


Prof.Adel Kamel 44

VI- ABDUCENT NUCLEUS

Abducent nucleus (GSE) is a small nucleus in the superior part of the floor of the 4th
ventricle near the midline beneath the facial colliculus. It supplies the lateral rectus muscle.

VII- FACIAL NERVE NUCLEI

A) Motor nucleus:(SVE); present deep in the reticular formation of the caudal pons, posterior
to the trapezoid body and medial to the spinal tract of the trigeminal nerve. The facial motor
nucleus innervates facial muscles.

Connections
- The part of the motor nucleus supplying the upper facial muscles receives corticobulbar
tracts from both sides, while the part of the nucleus supplying the lower part of the face
receives corticobulbar fibers from the opposite cerebral hemisphere.

B) Parasympathetic nuclei (GVE)


These nuclei lie posterolateral to the motor nucleus in caudal pons. They are referred to as the
superior salivatory nucleus and the lacrimal nucleus. The lacrimal nucelus has preganglionic
parasympathetic fibers that pass with the greater petrosal nerve relaying in the pterygopalatine
ganglion and the postganglionic fibers supply secretomotor fibers to the lacrimal, pharyngeal,
nasal and palatine glands. Fibers from the superior salivatory nucleus pass in the chorda
tympani relaying in the submandibular ganglion to supply secretomotor fibers to the
submandibular and sublingual salivary glands. This nucleus receives afferents from the hypo-
thalamus, olfactory system and nucleus solitarius. These connections serve the salivary reflex.
C) Gustatory nucleus (SVA) is the superolateral enlarged part of the nucleus solitarius. The
nucleus solitarius is present deep in the medulla anterolateral to dorsal motor nucleus of
vagus. The gustatory nucelus receives taste sensation from the facial, glossopharyngeal and
vagus nerves. The first order neuron of taste of the facial nerve is present in the geniculate
ganglion. Gustatory nucleus projects to the thalamus (PMVNT), hypothalamus, reticular
formation and other cranial nerve nuclei. The non-gustatory part of the nucleus solitarius
receives general visceral afferents from structures innervated by the glossopharyngeal nerve
(pharynx) and vagus nerve (oesophagus and abdominal viscera). It, also, receives afferents
from the spinal cord, cerebellum and cerebral cortex. This part of the nucleus solitarius
projects to reticular formation (which is implicated in the central regulation of respiratory,
cardiovascular and emetic functions.
Prof.Adel Kamel 45

VIII- VESTIBULAR NUCLEI

The vestibular nuclei (SSA) are complex comprise the following:


A) Medial vestibular nucleus lies under the vestibular area of the floor of the 4th ventricle
extending from the medulla to the pons.
B) Lateral vestibular nucleus is anterolateral to the upper part of the medial nucleus. It has
large cells and its upper end is continuous with lower end of superior vestibular nucleus.
C) Superior vestibular nucleus extends higher in the pons than the other vestibular nuclei
occupying the upper part of the vestibular area.
D) Inferior vestibular nucleus lies lateral to the medial nucleus and reaches to a lower
medullary level.

Vestibular pathways
1-Vestibular receptors : hair cells in cristae ampullaris detect angular movements of the head.
Hiar cells in mcula utriculi and macula sacculae detect linear head movements. Vestibular
nerve fibers reach bipolar neurons of the vestibular ganglion.
2-Vestibular ganglion is located at the distal end of the internal auditory meatus close to the
cristae and maculae. It contains bipolar neurons; whose distal processes receive vestibular
impulses fro hair cells, and whose central processes form vetibular nerve that joins the
cochlear nerve to form the 8th cranial nerve. The 8th cranial nerve travels with the facial nerve
in the internal auditory canal. After crossing the subarachnoid space CN VIII attaches to the
pontomedullary junction near the lateral recess of the 4th ventricle. Intraaxially the vestibular
nerve splits ventrally to rach the vestibular nuclei.
3-Vestibular nuclei represent the second order neuron of the vestibular pathway.
4- Efferents from vestibular nuclei (mainly medial nucleus) project to the inferior part of the
PMVNT and then to the vestibular area found in the superior temporal gyrus (in front of the
primary auditory area) and in the lower primary sensory area.
5-Connections of the vestibular nuclei (fig.2-11):
-Cerebellum; cerebellovestibular and vestibulocerebellar fibers with the flocculonodular lobe
(lobe concerned with balance). These connections are with the superior and lateral nuclei.
-Medial longitudinal bundle (MLB) receives from the superior and the medial vestibular
nuclei connecting them with ocular nuclei (III, IV & VI). By this the vestibular system
coordinates movements of the head with eye movements. MLB descends to the spinal cord as
sulcomarginal tract (medial vestibulospinal tract).
-Spinal cord; the large cells of the lateral nucleus project the vestibulospinal tract which is
concerned with activation of extensor (antigravity) muscles.
-Reticular formation: connects vestibular nuclei with the vomiting and vasomotor centers.
-Afferents from the inner ear (utricle, saccule and semicircular canals) reach the vestibular
nuclei through the vestibular nerve.
Prof.Adel Kamel 46

Fig.2-11. Connections of the vestibular nuclei.

Functions of the vestibular system


1-Detection of head position and movements, providing a sense of balance (other systems
contributing to balance are the musculoskeletal proprioceptors, visual and auditory systems).
2-Mediation of vestibule-ocular reflexes, counter-roll the eyes against movements of the head.
Eyes remain on visual target despite head movements.
3-Activation of antigravity muscles, to support the skeleton against collapse by pull of
gravity.
Prof.Adel Kamel 47

Clinical notes
A)An acute lesion of the vestibular system causes:
1-Vertigo with falling to the side of lesion, and inability to maintain vertical posture.
2-Nystagmus (rhythmic to-and-fro eye movements, oscillopia (apparent to-and-fro
movements of objects), and blurred vision.
3-Autonomic symptoms and signs (palor, sweating, nausea, vomiting and hypotension).
B)Motion sickness produces features associated with vestibular nerve lesion.
C)Meniere's sickness is recurrent vestibular symptoms as well as hearing deficits and tinnitus
(cochlear nerve involvement). The same manifestations occur in labyrinthitis as a result of
viral, bacterial or toxic causes (alcohol, salicylates).
D)Overactivity of antigravity muscles is a symptom of decerebrate rigidity which results form
transaction of the brain stem between the red nucleus and vestibular nuclei. This rigidity can
be abolished by sectioning of the vestibular nerves.
E)MLB lesion results in medial rectus paresis on attempt to lateral gaze associated with
horizontal nystagmus.

VIII- COCHLEAR NUCLEI (SSA)


The cochlear nuclei are two in number; anterior and posterior. The anterior nucleus is
present in the lower part of the pons on the anterolateral aspect of the inferior cerebellar
peduncle. The posterior cochlear nucleus is present in the lower pons on the posterior aspect
of the inferior cerebellar peduncle lateral to the vestibular area. These nuclei represent the
second order neuron in the cochlear pathway. The first order neuron is cells of the spiral
ganglion of the cochlea (see the auditory pathway).

IX- GLOSSOPHARYNGEAL NUCLE

A) Gustatory nucleus (SVA) receives taste from the posterior third of the tongue and the
valleate papillae. The first order neuron of this SVA is the ganglion of the glossopharyngeal
nerve. General sensations from the posterior third of the tongue carried by the IX nerve join
the spinal nucleus of the trigeminal nerve. Impulses from the carotid sinus terminate in the
dorsal motor nucleus of vagus nerve.. GVA from pharynx end in the nucleus solitarius.
Prof.Adel Kamel 48

B) Motor nucleus (SVE) is the upper end of the nucleus ambiguus. It supplies the
stylopharyngeus muscle and the upper pharyngeal constrictors. The nucleus ambiguous is
located deep in the reticular formation of the medulla between the spinal nucleus of the
trigeminal nerve and the inferior olive. It extends from the level of sensory decussation to the
upper level of the medulla. The nucleus ambiguous is composed of multipolar lower motor
neurons whose axons innervate the pharynx and larynx. The nucleus receives corticobulbar
fibers from both sides.

C) Inferior salivatory nucleus (GVE) is located in the reticular formation of the medulla
below the superior salivatory nucleus. It carries parasympathetic fibers via tympanic branch of
the glossopharyngeal nerve to lesser petrosal nerve which relays in otic ganglion and the
postganglionic parasympathetic fibers are carried by the auriculotemporal nerve to the parotid
gland. The inferior salivatory nucleus receives afferents from the hypothalamus through the
descending autonomic pathways. Information concerning taste is received from nucleus
solitarius.

X- VAGUS NERVE NUCLEI

A)Lower part of gustatory nucleus (SVA) receives taste from the epiglottis.

B)Nucleus solitarius (GVA); the lower non-gustatory part of the nucleus solitarius receives
general visceral afferents from the tongue, palate and pharynx. The vagus nerve contains
somatic afferent fibers (GSA from external ear) which terminate in the spinal nucleus of V.

C)Nucleus ambiguous (SVE) supplies pharyngeal constrictors and intrinsic muscles of


larynx.

D)Dorsal motor nucleus (GVE & GVA): the dorsal motor nucleus of the vagus is sited in the
central gray matter and extends from the middle to the upper level of the medulla
posterolateral to the hypoglossal nucleus. Its motor fibers are parasympathetic distributed to
involuntary muscles of bronchi, heart, oesophagus, stomach, intestine and part of the colon. It
receives afferents from the hypothalamus (autonomic pathway) and the glossopharyngeal
nerve (carotid sinus reflex). The dorsal motor nucleus of the vagus nerve receives afferent
sensory fibers (GVA) from oesophagus and abdominal viscera and project them to nucleus
solitaries.
Prof.Adel Kamel 49

XI- ACCESSORY NERVE NUCLEI

A) Nucleus ambiguous (SVE) gives the cranial accessory which is distributed in the
pharyngeal, palatal and recurrent laryngeal branches of the vague nerve.

B) Anterior horn cells (GSE) of the upper 5 cervical spinal cord segments give the spinal
accessory which supplies the sternomastoid and trapezius muscles. The spinal nucleus of the
accessory nerve receives corticospinal fibers from both hemispheres.

XII- HYPOGLOSSAL NUCLEUS

The hypoglossal nucleus (GSE) is 2 cm long, its upper part corresponds with the hypoglossal
triangle in the floor of the 4th ventricle; its lower part extends into the closed medulla. The
nucleus receives corticobulbar fibers of both sides but the part supplying genioglossus muscle
receives mainly from opposite side. The hypoglossal nucleus is connected to nucleus
solitarius and the medullary reticular formation.
Prof.Adel Kamel 50

RETICULAR FORMATION
Definition and extent. RF consists of widely spaced neurons, loosely arranged into nuclear
groups throughout the brain stem. Huge dendritic trees and immense highly collateralized
afferent and efferent network of axons cause the reticular appearance. The RF has the most
heterogeneous connections of the CNS. It receives impulses from and sends impulses to all
parts of the CNS; thus it influences all motor, sensory and mental function. RF extends
caudally to the medullocervical region, and rostrally to the midbrain-diencephalic junction.
The diffuse network is divided into three longitudinal columns (fig.2-12):
1-Median column (raphe nuclei) formed of intermediate neurons.
2-Medial column formed of large neurons (gigantocellular nuclei).
3-Lateral column formed of small neurons (parvocellular nuclei).
The groups of neurons are poorly defined. However polysynaptic pathways exist and both
crossed and uncrossed ascending and descending pathways are present, involving many
neurons that serve both somatic and visceral functions. Inferiorly the reticular formation is
continuous with the interneurons of the gray matter of the spinal cord where superiorly
impulses are relayed to the whole cerebral cortex. Substantial projections of fibers also leave
the reticular formation to the cerebellum.

Connections of the reticular formation


RF receives collaterals from most ascending and descending pathways. The RF has wide and
extensive efferent projections to most parts of the central nervous system.

Fig.2-12.Dorsal view of brainstem showing location of nuclei of reticular formation.


Prof.Adel Kamel 51

Functions of the reticular formation


1-RF controls motor activity through the reticulobulbar and reticulospinal tracts. The RF can
influence the activity of the alpha and gamma motoneurons. The reticular formation can
modulate the muscle tone and reflex activity. Stimulation of the gigantocellular medullary RF
tends to inhibit somatomotor activity whereas stimulation of the pontine gigantocellular RF
tends to facilitate somatomotor activity. RF can also bring about reciprocal inhibition; for
example, when the flexor muscles contract the anatgonistic extensors relax.
2-RF maintains the tone of antigravity muscles assisted by the vestibular apparatus. The
pontine reticulospinal tract terminates in laminae VII and VIII acting on axial muscles to
mediate postural reflexes and muscle tone.
3-RF controls somatic and visceral sensations. It influences the ascending pathways that
pass to supraspinal centers by facilitatory or inhibitory mechanisms. It has a role in gating
"mechanism" for the control of pain especially raphe and periaqueductal nuclei.
4-Higher control of autonomic nervous system from the cerebral cortex, hypothalamus and
other subcortical areas can be exerted by the reticulobulbar and the reticulospinal tracts which
synapse to the sympathetic and parasympathetic outflows.
5-RF controls the endocrine system; either directly or indirectly through the hypothalamic
nuclei. RF can influence the synthesis or secretion of releasing or release-inhibiting factors
and thereby control the activity of the pituitary gland.
6-RF influences the biological clocks, by means of its afferent and efferent pathways to the
hypothalamus.
7-Respiratory and cardiovascular centers are present in the medullary and pontine reticular
formation.
8-The reticular activating system is an ascending pathway carrying information to the
cerebral cortex. This causes a sleeping person to awaken. Different degrees of wakefulness
depend on the degree of activity of the reticular formation. The state of consciousness
depends on the continuous projection of sensory information to the cortex. RF controls the 4
As of sleep-wake state; Asleep-Awake-Alert-Attentive.
9-Homeostasis and neurovegetative reflexes; controlling breathing, pulse and blood pressure,
gastrointestinal activity, genitourinary motility and electrolyte balance.
-RF circuitry mediates reflexes of the respiratory and gastrointestinal tract through afferents
from V, IX, X cranial nerves. Reflexes include coughing, sneezing, swallowing, gagging, and
vomiting.

Clinical Notes
1-Destruction of the rostral pontine and midbrain RF (essential for arousal, alert and
consciousness state) causes transitory loss of consciousness.
2-Lesion of the caudal pontine and medullary RF (essential for automatic breathing, and
cardiovascular reflexes) causes respiratory dysrhythmia, hypotension and Horner's syndrome.
Prof.Adel Kamel 52

STUDY QUESTION ON CHAPTER 2


Choose ONLY one correct answer
1-False about spinal trigeminal nucleus 6-Supplies laryngeal muscles
a)It is homolog of substantia gelatinosa. a)Nucleus ambiguous.
b)It projects to contralateral PMVNT. b)Nucleus solitaries.
c)It is superficial to trigeminal tract. c)Dorsal motor nucleus of vagus.
d)It carries pain sense from the face. d)Inferior salivary nucleus.

2-Posterior external arcuate fibers arise from 7-Lateral medullary syndrome is


a)Accessory cuneate nucleus. due to occlusion of
b)Arcuate nucleus. a)Anterior spinal artery.
c)Hearing pathway. b)Posterior spinal artery.
d)Gracile nucleus. c)Posterior inferior cerebellar A.

3-Dorsal tegmental decussation is d)Pontine arteries.


a)Decussation of tectospinal tracts. 8-Is not a component of crus cerebi
b)Decussation of rubrospinal tracts. a)Corticobulbar tract.
c)Decussation of internal arcuate fibers. b)Corticospinal tract.
d)Decussation of olivocerebellar fibers. c)Frontopontine fibers.
4-Cranial nerve which does not possess d)Rubrospinal tract.
Five axonal components is 9-Vestibular ganglion is in
a)Facial nerve. a)Modulus.
b)Glossopharyngeal nerve. b)Distal internal auditory meatus.
c)Vagus nerve. c)Close to pontomedullary junction.
d)Oculomotor nerve. d)At lateral recess of 4th ventricle.

5-Is not a sign of vestibular neuritis 10-Inferior salivary nucleus is in


a)Vertigo. a)Caudal pons.
b)Nystagmus. b)Rostral medulla.
c)Disturbed posture. c)Caudal midbrain.
d)Dryness of the eye. d)Middle pons.
Prof.Adel Kamel 53

CHAPTER 3

CEREBELLUM
Objectives
-Definition and phylogeny of cerebellum.
-External features of the cerebellum.
-Composition and internal structures.
-Cerebellar input and output.
-Anatomical basis of cerebellar lesions

Definition, position and function: The cerebellum (little brain) is the largest part of the
hindbrain, 150 gm in weight.. It is located in the posterior cranial fossa covered by the
tentorium cerebelli. Supported by its peduncles, the cerebellum overhangs the superior
and inferior medullary vela (the roof of the fourth ventricle). The function of the
cerebellum is entirely motor. It maintains balance, influences posture and muscle tone and
it coordinates movements.
External features

The cerebellum is formed of two cerebellar hemispheres joined by a narrow median zone
called the vermis. Anterior and posterior notches are present as deficiencies anteriorly and
posteriorly between the two hemispheres.

Surfaces

On the superior surface the superior vermis can not be demarcated from the superior
surface of the hemisphere. The vermis seen on the inferior surface is called inferior vermis
and is separated from the inferior aspect of the cerebellum by deep median hollow termed
vallecula cerebelli. The inferior vermis consists of three small lobules named nodule,
uvula and pyramid.
Prof.Adel Kamel 54

Cerebellar fissures and lobes (fig.3-1)

The cerebellar surface is highly convoluted with the folds or folia separated by numerous
transverse fissures. Fisures vary in depths and some are used as landmarks to divide the
cerebellum into lobes. Only three deep fissures are of significance:

Horizontal fissure sweeps from the anterior to the posterior notches. It separates the
superior and inferior surfaces of the cerebellum. The anterior part of the fissure receives
the middle cerebellar peduncle (laterally located) with which both the superior (medially
located) and inferior (middle located) cerebellar peduncles are closely assossiated.

Primary fissure is a V-shaped fissure cutting the superior vermis at the junction of its
anterior 2/3 with its posterior 1/3. The fissura prima curves laterally to meet the horizontal
fissure. The part in front of primary fissure is termed the anterior lobe; the part behind is
the middle lobe.

Posterolateral fissure is present on the undersurface of the cerebellum separating the


flocculonodular lobe from the rest of the cerebellum.

Fig.3-1.Diagram of the cerebellar fissures and lobes.


Prof.Adel Kamel 55

Composition: The cerebellum consists of:

1-Cerebellar cortex
The cerebellar cortex is formed of three layers (fig.3-2):

1-Molecular layer
It is the outer layer that underlies the pia mater. It contains dendritic arborizations of
Purkinje cells, axons of granule cells, and stellate and basket cells. Stellate and basket cells
make inhibitory synapses with Purkinje dendrites to limit the area of the cortex excited by
climbing and mossy fibers.
2-Purkinje cell layer: Dendrites of Purkinje cells are flattened in the molecular layer.
Their axons make inhibitory synaptic contacts on neurons of the deep cerebellar nuclei
which originate most of cerebellar efferent fibers.
3-Granule cell layer contains granule cells, Golgi type II cells and cerebellar glomeruli.
Fibers enter the cerebellum as clmbing fibers (originate from inferior olivary nucleus) and
mossy fibers (all other afferents). Mossy fibers are excitatory fibers which synapse with
granule cell dendrites in the form of rosettes. Each rosette forms the core of a glomerulus.
Each axon of mossy fibers may affect thousands of granule cells. Dendrites of granule
cells, and dendrites and axons of Golgi type II cells synapse with mossy fibers. The output
is via axons of granule cells. Unmyelinated axons of granule cells ascend into the
molecular layer where they divide and run parallel to long axis of folia. They run at right
angles to and through dendritic expansions of Purkinje cells, making excitatory synaptic
contacts with several Purkinje cells. Climbing fibers are excitatory terminating on
cerebellar nuclei and on dendrites of Purkinje cells. Relationship of climbing fibers and
Purkinje cells is one to one.
It appears that all afferent pathways ultimately converge on Purkinje cells. Purkinje axons
are the only way out of the cerebellum. Purkinje axons send recurrent collaterals back to
synapse with adjacent Purkinje cells and to Golgi Type II neurons.
2-Deep white matter is made up of afferent and efferent fibers that run to and from the
cerebellar cortex.
3-Three pairs of deep nuclei (fig.3-3) which receive afferents and are the primary source
of efferent fibers from the cerebellum to the other parts of the brain. From medial to lateral
the cerebellar nuclei are:
a)Fastigial nucleus.
b)Nucleus interpositus, formed of globose and emboliform nuclei
c)Dentate nucleus.
4-Three pairs of peduncles that convey afferent and efferent fibers. They are inferior,
middle and superior cerebellar peduncles, which join the cerebellum to the medulla, pons
and midbrain, respectively.
Prof.Adel Kamel 56

Fig.3-2.Internal structure of the cerebellum.

Fig.3-3.Cerebellar nuclei.
Prof.Adel Kamel 57

Phylogeny (developmental history) of the cerebellar lobes (fig.3-1)

1-Archicerebellum (flocculonodular lobe or vestibulocerebellum) is the original part of


the cerebellum concerned with maintenance of balance. It has extensive connections
(afferents and efferents) with the vestibular and reticular nuclei. The influence of the
archicerebellum upon the lower motor neurons is mediated by means of vestibulospinal
and reticulospinal tracts. Lesion of this lobe (caudal vermis syndrome) produces
disturbance in equilibrium and trunk ataxia (patient walks as drunk and is unable to
maintain upright posture).

2-Paleocerebellum (anterior lobe or spinocerebellum) is phylogenetically intermediate.


Paleocerebellum influences muscle tone and posture. Afferents are mainly the dorsal and
ventral spinocerebellar tracts that carry information from muscles, joints and cutaneous
receptors to the cerebellar cortex of the ipsilateral vermis and paravermal zones. Efferents
pass to the nucleus interpositus (globose and emboliform nuclei) which in turn project to
inferior olivary nucleus and contralateral red nucleus of the midbrain. Red nucleus sends
the decussated rubrospinal tracts which facilitates flexor muscle tone. Lesion of this lobe
(rostral vermis syndrome) causes hypotonia, and trunk and leg ataxia.
3- Neocerebellum (posterior lobe, or cerebrocerebellum). With evolution of the
neopallium in mammals, further cerebellar expansion with addition of a larger middle lobe
forms between the archicerebellum and the paleocerebellum. The neocerebellum is
concerned with muscular coordination including force, speed and curved movements.
Afferents are principally pontocerebellar fibers which are influenced by widespread
regions of the cerebral cortex involved in planning and execution of movement.
Pontocerebellar fibers decussate and by means of middle cerebellar peduncle terminate in
the lateral zones of the contralateral cerebellar hemisphere. Efferents from the
neocerebellar cortex are directed to the dentate nucleus which in turn project to the
contralateral ventral lateral nucleus of the thalamus and red nucleus (dentatothalamic and
dentatorubral fibers). The ventral lateral nucleus of the thalamus projects to the cerebral
cortex, particularly the motor cortex of the frontal lobe. The neocerebellum thus exerts its
coordinating role in movement through the action on cerebral cortical areas giving rise to
corticnuclear, and corticospinal pathways. Lesion of this lobe causes hypotonia, pendulum
muscle jerks, and incoordination of the upper limbs (dysdiadochokinesia and intention
tremors), lower limbs (cerebellar ataxia), and speech (dysarthria), and eyes (nystagmus).

Blood supply of cerebellum


Three pairs of cerebellar arteries for superficial anastomoses:
1-Posterior inferior cerebellar arteries (vertebral arteries).
2-Anterior inferior cerebellar arteries (basilar artery).
3-Superior cerebellar arteries (basilar artery).
Veins drian in great cerebral vein or in adjacent venous sinuses.
Prof.Adel Kamel 58

Cerebellar peduncles
On each side of the pons, three peduncles- the superior, middle, and inferior unite into one
stalk of fibers that attach the cerebellum to the pons. On entering the cerebellum the
peduncles are oriented superior, inferior and middle from medial to lateral. The superior
cerebellar peduncle represents the major output of the cerebellum. The inferior cerebellar
peduncle is largely afferent. The middle peduncle is only afferents conveying the
pontocerebellar fibers.

Afferents and efferents of the cerebellar peduncles


_______________________________________________________________________

Peduncle Afferents Efferents


_______________________________________________________________________

Superior peduncle -Tectocerebellar T(conveys -Dentatorubral fibers.


visual,&auditory information -Dentatothalamic fibers.
to cerebellum).
-Ventral spinocerebellar tract.

-Hypothalamocerebellar fibers.
-Cereuolocerebellar fibers.
(Noradrenergic projection).

Middle peduncle -Pontocerebellar of opposite side. --------------------

Inferior peduncle
Juxtarestiform body-Vestibulocerebellar fibers. -Cerebellovestibular fibers.
Restiform body -Olivocerebellar tract. -Cerebelloolivary fibers.
-Cerebelloreticular fibers. -Reticulocerebellar fibers.

-Arcuatocerebellar.
-Trigeminocerebellar T.
-Dorsal spinocerebellar tract.
Prof.Adel Kamel 59

Cerebellar circuits
The cerebellum receives and sends well-defined afferent and efferent pathways.
1-Afferent pathways arise in the sensory system, motor cortex and reticular formation.
Arriving through one of the peduncles, they send collaterals to cerebellar nuclei and
proceed to reach the cerebellar cortex. Direct inputs are vestibulocerebellar,
spinocerebellar, trigeminocerebellar, olivocerebellar and corticopontocerebellar. Indirect
routes modifying cerebellar activity converge to sources of direct pathways. They include
other spinal routes (accessory olivary nuclei and reticular formation), subcortical nuclei
and cerebral cortex. Nevertheless, spinal activity may reach the cerebellum in different
orders of complexity. Direct spinocerebellar fibers may carry simple information about
pattern of single sensation; others carry patterns formed by convergence and interaction of
connections which may be modified by descending pyramidal tract. Indirect spinal paths
mediate integration of spinal impulses with those converging by other routes (RF and
accessory olivary nuclei) before entering the cerebellum. Cerebral neocortex modifies
cerebellar activity by corticopontocerebellar, corticotectal, corticoreticular and
corticoolivary tracts. Brain stem nuclei projecting to the cerebellum may be sites for pre-
integration of input along afferents to olivary nuclei (basal ganglia, RF and red nucleus).
2-Efferent pathways
The cerebellar nuclei are the main source of efferent fibers from the cerebellum to other
parts of the brain. Their output is mainly excitatory. Purkinje cells utilize GABA as their
neurotransmitter and their axons (the only axons to leave the cerebellar cortex) carry
inhibitory synaptic contacts to the cerebellar nuclei
a) Purkinje cells of the median vermian cortex project to fastigial nuclei which connect to
vestibular nuclei. The vestibulospinal tract influences the extensor muscle tone. Vermis of
the anterior lobe represents the body in an inverted manner with the feet most rostral and
arms most caudal. Lesion of rostral vermis is manifested by leg dystaxia.
b) The paramedian (paravemian) zone projects to nucleus interpositus which send efferents
to the red nucleus and inferior olivary nucleus. Decussated rubrospinal tract facilitates
flexor muscle tone.
c) The lateral cortical cerebellar zone projects to the dentate nucleus which sends efferents
to the red nucleus and thalamus (ventral lateral nucleus). Through rubrothalamic and
thalamocortical projections, this pathway assists in coordination of the willed movements.
All incoming climbing and mossy fibers produce excitatory synapses. Of the intrinsic
neurons, only the granule cell axons produce excitation. Inhibitory modulation of
intracortical circuitry is done by basket and stellate neurons. The most important cerebellar
circuit connects the cerebellum with the cerebral motor cortex. It consists of cerebro-
ponto-cerebello-dentato-thalamo-cortioc-pyramido-LMN circuit.
Prof.Adel Kamel 60

Decussations and laterality of cerebellar signs


Three major decussations take place in the cerebrocerebellocerebral circuits:
a) Pontocerebellar (in the middle cerebellar peduncle).
b) Dentatothalamic.
c) Dentatorubral (in the superior cerebellar peduncle).
These decussations bring coordinating influence of one cerebellar cortex on the
contralateral motor cortex. The pyramidal tract decussates before reaching the LMN. That
is why a cerebral lesion causes contralateral motor signs and a cerebellar hemisphere lesion
causes ipslateral motor signs. Damage of dentatothalamic pathway before it decussates
causes ipsilateral signs; after it decussates it causes contralateral signs.

Summary of the cerebellum

Review cerebellar triads:


-Cerebellum is formed of 3 phylogenetic lobes; flocculonodular (archicerebellum),
anterior lobe (paleocerebellum) and posterior lobe (neocerebellum).
-Cerebellum has 3 main fissures; primary, posterolateral and horizontal.
-Cerebellar cortex is formed of 3 layers; molecular, Purkinje and granular layers.
-White mater contains 3 pairs of nuclei; fastigial, interpositus and dentate nuclei.
-Cerebellum is longitudinally organized into 3 zones; vermian, paravermian and lateral
zones.
-Cerebellar peduncles are 3; inferior, middle and superior.
-Major cerebro-cerebellar decussations are 3; pontocerebellar, dentatothalamic and
dentatorubral.
-Cerebellar neurotransmitters are 3; GABA (utilized by Purkinje cells and is inhibitory),
glutamate (used by granule cells and is excitatory) and aspartate (used by climbing fibers
and is inhibitory or excitatory).
-Cerebellar functions are 3; maintain balance, influences muscle tone and coordinates
movements.
-Cerebellar dysfunctions present 3 main signs; hypotonia, dysequilibrium and
dyssynergia (dysarthria, dysmetria, nystagmus and intention tremors). A midline lesion
(tumor) leads to loss of postural control; as a result it is impossible to stand or sit without
toppling over. Unilateral lesion of a cerebellar hemisphere causes unsteady gait and
intention tremors on the same side of the body. Bilateral lesion or cerebellar ataxia (alcohol
intoxication, degenerative disease or multiple sclerosis) causes slow slurred speech,
unsteady (drunk) gait and incoordination of both arms.
Prof.Adel Kamel 61

STUDY QUESTIONS ON CHAPTER 3


Choose the correct answer
1-Is not a content of inf.cerebellar peduncle
a)Vestibulocerebellar fibers.
b)Posterior spinocerebellar fibers.
c)Cereuolocerebellar fibers.
d)Cuneocerebellar fibers.

2-Is not a sign of cerebellar dysfunction


a)Hypotonia.
b)Slurred speech.
c)Resting tremors.
d)Imbalance.

3-Axons in sup.cerebellar peduncle arise from


a)Deep nuclei of cerebellum.
b)Stellate neurons in cerebellar cortex.
c)Purkinje cells.
d)Glogi type II cells.

4-Cell that is excitatory in cerebellar cortex


a)Purkinje cell.
b)Granule cell.
c)Glogi type II.
d)Basket cell.

5-Which lesion wouldn't reduce afferents


To cerebellar nuclei
a)Transection of inf.cerebellar peduncle.
b)Section of the cochlear nerve.
c)Destruction of inferior olivary nucleus.
d)Section of superior cerebellar peduncle.
Prof.Adel Kamel 62

CHAPTER 4

SPECIAL SENSORY PATHWAYS


Objectives
-Trace order neurons of special sensory pathways (smell, vision, hearing and taste).
-Effects of interruptions of special sensory pathways.

OLFACTORY PATHWAY
Olfactory receptor cells are first order neuron and present in the olfactory mucosa in the
upper part of the nasal cavity. Central processes of these bipolar cells form olfactory
nerve fibers that pass through cribriform plate of ethmoid to enter the olfactory bulb
where olfactory nerve fibers synapse with mitral cells (second order neuron) and granular
cells. The olfactory bulbs are interconnected through the anterior commissure. From the
posterior end of the olfactory bulb runs back the olfactory tract which is formed of the
central processes of the mitral cells and fibers from opposite olfactory bulb. As the tract
reaches the anterior perforated substance it divides into medial and lateral olfactory
striae dispersing to a number of structures around the vallecula, roof of which is the
anterior perorated substance. The lateral olfactory stria enters primary olfactory area
which is the periamygdaloid and prepiriform areas. The medial olfactory stria carries
fibers that cross to the opposite olfactory bulb through the anterior commissure.

The cortex of the base of the temporal lobe expands forming pear-shaped structure known
as piriform lobe containing the piriform cortex which is the major primary olfactory cortex.
Caudally the medial part overlies amygdaloid complex and is referred as periamygdaloid
area, and further caudally is the parahippocampus. The primary olfactory cortex projects to
the thalamus (dorsomedial nucleus), amygdaloid nucleus, hypothalamus and entorhinal
area (secondary olfactory area). The primary olfactory cortex sends connections to centers
within the brain to establish emotional and autonomic responses to olfactory sensations.
The primary and secondary olfactory areas are responsible for subjective appreciation of
olfactory stimuli. Olfactory pathway (fig.4-1) is unique among sensations in that it reaches
the cortex without synapse with the thalamic nuclei. It is also unique in having only two
neurons between sensory receptors and the cerebral cortex.
Prof.Adel Kamel 63

Fig.4-1. Olfactory pathway.


Prof.Adel Kamel 64

VISUAL PATHWAY

Retina

Three neurons are involved in the retina; (1) rods and cones (receptors for light), (2)
bipolar neurons and (3) ganglion cells . Axons of ganglion cells form the optic nerve.
Cones concentrate at the macula and fovea centralis, and are responsible for acuity and
color vision. Fovea centralis lies within macula lutea lateral to the optic disc. It is the site
of highest visual acuity. Rods occupy peripheral retina and are responsible for night vision,
motion detection and registration of the periphery of visual field around the macula. If light
strikes the nasal half of the retina, it is interpreted that the object is located in the temporal
half of visual field. Superior part of an object is projected to the inferior half of the retina.
The actual retinal image is a real inverted image. Human eyes angle forward viewing
objects by means of stereoscopic binocular vision. The axonal arrangement must integrate
the nasal half of the visual field of one eye with the temporal half of the visual field of the
other eye. This representation requires a shift from total decussation of the optic chiasm
(primitive arrangement) to partial decussation (advanced arrangement). Eyes of lower
mammals and submammalia angle to the sides. These animals view an object by
panoramic vision, in which the fields of the two eyes do not overlap. In order to represent
the fields from their two eyes as a continuous panoramic field, the optic nerve fibers
undergo total decussation.

Optic nerve

The optic nerve unites with the optic nerve of the opposite side to form the optic chiasma.

Optic chiasma

The optic chiasma is present at the floor of the third ventricle above the pituitary gland. In
the optic chiasma, fibers of the nasal half of each retina cross the midline and enter the
optic tract of the opposite side, while fibers from temporal half of each retina pass
posteriorly in the optic tract of the same side (fig.4-2).
Prof.Adel Kamel 65

Optic tract

The optic tract passes posterolaterally around the hypothalamus and crus cerebri. Most of
fibers terminate by synapsing with neurons of the lateral geniculate body of the thalamus.

Lateral geniculate body (LGB)

The lateral geniculate body projects from the pulvinar of the thalamus. Few fibers from the
optic tract pass through the brachium of the superior colliculus to the pretectal nucleus and
the superior colliculus, and are concerned with light reflexes. Axons of cells of the LGB
form the optic radiation.

Optic radiation

The optic radiation passes posteriorly in the retrolentiform part of the internal capsule and
terminate in the visual areas in the occipital lobe. Fibers representing lower 1/2 of the
visual field terminates in the upper visual cortex (above calcarine sulcus). Fibers
representing upper 1/2 of the visual field sweep into the temporal lobe forming Meyer's
loop to terminate in the visual cortex below calcarine sulcus.

Visual areas

The primary visual area 17 occupies the upper and lower lips of the calcarine sulcus on
the medial surface of the occipital lobe. The secondary visual area (visual association
area 18,19) relates the visual information received by area 17 to previous visual
experiences, thus enabling the individual to recognize and appreciate what he/she is seeing.
The secondary visual area contains also the occipital eye field responsible for reflex
conjugate deviation of the eyes to the opposite side.

Blood supply of the visual pathway

1-Optic nerve: cntral retinal artery, recurrent branch of ophthalmic artery, posterior ciliary
artery and superior hypophysial artery.
2-Optic chiasma: anterior cerebral artery, internal carotid artery (via stalk of pituitary).
3-Optic tract: anterior choroid and posterior communicating.
4-Optic radiation: deep branches of middle and posterior cerebral arteries.
Prof.Adel Kamel 66

Clinical notes

-The retinal areas have precise point-to-point relationship with the LGB, each portion of
the retina projects on a specific portion of the LGB. Fibers from the upper retinal quadrants
terminate in the medial half, those from the lower quadrants terminate in the lateral half.
The macular fibers occupy the central portion of the LGB.
-A similar point-to-point projection exists between the LGB and the visual cortex. The
medial portion of the LGB (lower visual field) projects to the superior lip of the calcarine
sulcus and fibers form the superior portion of optic radiation. The lateral portion of the
LGB (upper visual field) projects to the inferior lip of the calcarine sulcus. These fibers
occupy the inferior portion of optic radiation. The macular fibers which constitute the
intermediate part of the optic radiation, terminate in the posterior third of the visual cortex.
-The visual cortex receives fibers from temporal half of the ipsilateral retina and the nasal
half of the contralateral retina. The right half of the field of vision, therefore, is represented
in the left visual cortex and vice versa. This is called binocular vision.
-Frontal eye field (area 8) controls voluntary movements of the eye. For area 8 to be active
it receives information from area 17. Area 8 controls ocular nuclei through corticonuclear
fibers.
-Lesions of the visual pathway (fig.4-2)
Lesions at different sites along the visual pathway result in characteristic visual defects:
1-Lesion of one optic nerve results in blindness (anopia) of the same eye.
2-Lesion of both lateral angles of the optic chiasma results in binasal hemianopia.
3-Lesion of the central part of optic chiasma results in bitemporal heteronymous
hemianopia.
4-Lesion of the optic tract on one side results in loss of the opposite field of vision called
homonymous hemianopia. Lesion of the right optic tract results in left hemianopia.
5-Lesion of the lower part of the optic radiation in one side (left) results in (right) upper
quadrantanopia.
6-Lesion of the upper part of the optic radiation in one side (left) results in (right) lower
quadrantanopia.
7-Lesion of the lateral geniculate body or of the whole optic radiation results in
homonymous hemianopia.
8-Lesion of the primary visual cortex results in homonymous hemianopia with macular
sparing, i.e. preserve the central vision.
9-Lesion of the secondary visual areas leads to visual agnosia (inability to recognize seen
objects).
10-Lesion of the visual eye fileld (area 8) leads to conjugate eye movements to tne same
side of the lesion.
Prof.Adel Kamel 67

Fig.4-2.Visual pathways and effects of their lesions.


Prof.Adel Kamel 68

VISUAL REFLEXES

Light reflex

If one eye is stimulated by light, the pupils of both eyes normally constrict. Constriction of
the pupil of the stimulated eye is called the direct light reflex (fig.4-3); constriction of the
opposite pupil is called the consensual light reflex. Afferents travel through optic nerve,
optic chiasma and optic tract. Small number of fibers synapses with the pretectal nucleus
which lies close to the superior colliculus. Axons of the pretectal nucleus synapse with
Edinger-Westphal nuclei of oculomotor nerves of both sides. Crossing of fibers occur in
the posterior commissure close to the cerebral aquiduct. Parasympathetic fibers travel
through the oculomotor nerves to relay in the ciliary ganglia. Postganglionic parasym-
pathetic fibers pass through the short ciliary nerves to the constrictor pupillae muscle of the
iris producing pupillary constriction.

Accommodation reflex
Accommodation to near vision occurs by:
1-Convergence of the ocular axes by contraction of the medial recti.
2-Increase refractive power of the lens by contraction of the ciliary muscle.
3-Restriction of light to thick central part of lens by contraction of the sphincter pupillae.
Afferents travel through the optic nerve, optic chiasma, optic tract, LGB, and the optic
radiation to the visual cortex. The visual cortex is connected to the frontal eye field (area
8). Cortical fibers from area 8 descend through the internal capsule to the oculomotor
nuclei in the midbrain. Impulses travel to both medial recti producing their contractions
which results in convergence.
Some fibers synapse with the Edinger Westphal nuclei of both sides. Parasympathetic
fibers come out with the oculomotor nerve, relay in ciliary ganglion and short ciliary
nerves supply the ciliary muscle which contracts resulting in increase convexity and power
of the lens, and also short ciliary nerves supply the sphincter pupillae which contracts
resulting in pupillary constriction and restriction of light on the central thick part of the
lens.

Corneal reflex
Light touching of the cornea or conjunctiva results in blinking of the eye lids. Afferents
travel in the ophthalmic nerve, sensory nucleus of trigerninal nerve, internuncial neurons,
medial longitudinal bundle, motor nucleus of facial nerve, facial nerve to the orbicularis
oculi resulting in closure of the eye lids.
Prof.Adel Kamel 69

Visual body reflexes


Automatic movements of the eyes, head, and neck toward the source of visual stimulus and
the protective closing of the eyes or raising the arm for protection. Pathway includes optic
nerve, optic chiasma, optic tract, superior colliculi, tectospinal and tectonuclear tracts to
neurons of anterior horn cells and cranial nerve nuclei.

Reflex dilatation of the pubil


Stress or emotional excitements stimulate hypothalamus which through the reticular
formation stimulates the lateral horn cells of T1 and T2 segments that relay in the superior
cervical ganglion. Postganglionic sympathetic fibers pass through the internal carotid
plexus and the long ciliary nerves to the dilator pupillae muscle resulting in pupillary
dilatation.

Fig.4-3.Visual reflexes.
Prof.Adel Kamel 70

HEARING (AUDITORY or COCHLEAR) PATHWAY

-Organ of Corti is present in the cochlea. It has hair cell receptors on the tectorial
membrane which are tonotopically organized to detect sound of low to high frequencies.

-Spiral (cochlear) ganglion is located in the modiolus or core of the cochlea, and contains
bibolar auditory neurons. Distal branches of bibolar cells end in contact with the hair cells
located along the coiled cochlear duct. Hair cells transduce various frequencies of sound
into nerve impulses. Central processes of bibolar neurons form the cochlear nerve.

-Cochlear nerve runs centrally through the internal auditory meatus in company with the
vestibular nerve and facial nerve. After crossing the subarachnoid space, the cochlear nerve
enters the caudal pons at the cerebellopontine angle. Fibers of the cochlear nerve then
divide and terminate in the cochlear nuclei.

-Cochlear nuclei contain the second order neuron; are dorsal (posterior) and ventral
(anterior) nuclei present on the surface of the inferior cerebellar peduncle. Cochlear nuclei
send axons that run medially through the pons to end in nuclei of auditory pathway (fig.4-
4) of the same and opposite side. These auditory nuclei are: a) Nuclei of the trapezoid
body, b) Superior olivary nuclei, and c) Nuclei of lateral lemniscus.

Axons from the auditory nuclei form an interconnected trapezoidal configuration in the
caudal pontine tegmentum called trapezoid body.

-Efferent third and fourth order neuron pathways from the auditory nuclei

Most of fibers relayed in the trapezoid body and superior olivary nucleus cross to the
opposite side in the anterior pontine tegmentum and ascend as lateral lemniscus. Some
fibers ascend in the lateral lemniscus of the same side. This indicates the bilateral
representation of the hearing pathway. Axons of the lateral lemniscus terminate in; a)
Nucleus of the lateral lemniscus (third order neuron) which establishes reflex connections
with motor nuclei of the facial and trigeminal nerves mediating contraction of stapedius
and tensor tympani muscles in response to loud noise, b) Nucleus of the inferior
colliculus of the midbrain which relays auditory impulses to the medial geniculate body
Prof.Adel Kamel 71

via the brachium of inferior colliculus, c) Medial geniculate body (MGB) is the specific
thalamic sensory relay nucleus for hearing. It gives the geniculotemporal tract that projects
in the sublentiform part of the internal capsule (auditory radiation) to the auditory receptive
areas. Axons that synapse on the primary auditory cortex in a tonotopic order constitute the
core projection from the MGB. Axons that synapse on the secondary auditory cortex that
surrounds the primary auditory cortex constitute the belt projection from the MGB.

-The primary auditory area (areas 41 and 42) is present in the superior temporal gyrus
and largely hidden in the lateral sulcus in the transverse gyri of Heschl in the superior bank
of the lateral sulcus. The primary auditory cotex is concerned with reception of sounds.
Recognition and interpretation of sounds takes place in the secondary auditory area (area
22), in the posterior part of the superior temporal gyrus (Wernicke's sensory speech area).

Clinical notes

1-The tonotopic organization (high tone and low tone localization) present in organ of
Corti is preserved within the cochlear nuclei, the inferior colliculi and in the primary
auditory area.
2-Descending fibers originating in the auditory area and in other nuclei in the auditory
pathway accompany the ascending pathway. These fibers end on nerve cells at different
levels of the auditory pathway and on hair cells of organ of Corti, serving as feedback
mechanism that plays a role in auditory sharpening, suppressing some signals and
enhancing others.
3-Relay nuclei along the auditory pathway are involved in reflex connections by which
various motor phenomena occur in response to cochlear stimulation. Additional fibers
enter the brain stem reticular formation and are involved in reflex closing of the eyes and
turning of the head in response to loud noise. Inferior colliculi are interconnected and
project to superior colliculi. Certain neurons in inferior colliculus are concerned with
localization of the source of sound.
4-Destruction of cochlear nerve or both cochlear nuclei causes complete deafness on the
same side. Lesion of one lateral lemniscus or of the auditory cortex causes bilateral
diminution of hearing (partial deafness) that is most marked on the contralateral side.
Acoustic neuroma is a tumour of the 8th cranial nerve and leads to compression of the
nerve and adjacent structures in the cerebellopontine angle. Attacks of dizziness
accompanied by deafness occur. With expansion of the tumour, ataxia, paralysis of V and
VII cranial nerves, and the limbs follow.
5-The superior olivary nucleus mediates fibers of auditory pathway to relay on abducent
nucleus which associate hearing with lateral eye movements. Axons from the superior
olivary nucleus connect with motor neurons of stapedius (VII) and tensor tympani (V) that
modulate and dampen vibrations of bony ossicles thus protecting hair cells of organ of
Corti from damage by loud noise. Olivocochlear efferent tract projects back to organ of
Corti (crossed and uncrossed) which influences reception of sound waves at organ of Corti.
Prof.Adel Kamel 72

Fig.4-4.Hearing (Cochlear) pathway.


Prof.Adel Kamel 73

TASTE PATHWAY

Receptors of taste are taste buds present in the tongue, soft palate and epiglottis.

Taste of anterior 2/3 of the tongue is carried by chorda tympani of facial nerve, taste from
posterior 1/3 of the tongue is carried by the glossopharyngeal nerve, and taste from the
back of the tongue and epiglottis is carried by the superior laryngeal nerve of vagus. Taste
from soft palate is carried by the greater palatine nerve to sphenopalatine ganglion then to
the greater petrosal nerve. The first order neuron is the sensory ganglia of cranial nerves
VII, IX and X. The second order neuron is the gustatory nucleus present at the upper
part of the nucleus solitarius (fig.4-5). Nucleus solitaries sends two ascending pathways:

a)Through the central tegmental tract to PMVNT (non-lemniscal pathway to the thalamus).

b)To parabrachial nuclei of reticular formation. They are present around the superior
cerebellar peduncle and relay to the thalamus, hypothalamus and amygdaloid nucleus.

Fibers cross to the opposite side and ascend in the reticular formation to reach the third
order neuron which is the PMVNT. Posterior medial ventral nucleus of the thalamus
(PMVNT) project to the gustatory cortex. Taste area lies in the gustatory cortex of
parietal operculum (area 43) and parainsular cortex. Salivary reflex is mediated through
connections between nucleus solitarius and salivary nuclei. Ageusia (gustatory anesthesia)
is lack of the sense of taste which is common in heavy smokers.
Prof.Adel Kamel 74

Fig.4-4. Taste pathway.


Prof.Adel Kamel 75

STUDY QUESTIONS ON CHAPTER 4

Choose the correct answer


1-False about primary olfactory cortex
a)Receives input from lateral olfactory stria.
b)Projects to the thalamus.
c)Includes entorhinal cortex.
d)Includes prepiriform cortex.

2-Not a component of taste pathway


a)Geniculate ganglion.
b)Semilunar ganglion.
c)Nucleus solitaries.
d)Central tegmental tract.

3-Lesion of left optic tract results in


a)Left anopia.
b)Left hemianopia.
c)Right hemianopia.
d)Binasal hemianopia.

4-Lesion of right 1ry auditory area results in


a)Right deafness.
b)Left deafness.
c)Diminish mainly right hearing.
d)Diminish mainly left hearing.

5-False about fovea centralis


a)Contains only cones.
b)Lies within macula lutea.
c)Site of highest visual acuity.
d)Lies medial to optic disc.
Prof.Adel Kamel 76

CHAPTER 5

DIENCEPHALON

Objectives
-Orientation of the components of the diencephalons.
-Role of subthalamus in motor control.
-Relations of thalamus and classifications of thalamic nuclei.
-Connections, motor, sensory and memory functions and dysfunctions of the thalamus.
-Relations, classification of nuclei and significances of hypothalamus.

The diencephalon is formed of Epithalamus, Subthalamus, Metathalamus, Thalamus


and Hypothalamus.

EPITHALAMUS
Epithalamus consists of the habenular nuclei and their connections and the pineal gland.

Habenular nucleus is situated medial to the posterior surface of the thalamus. It


receives afferents from septal nuclei, reticular formation and amygdaloid nucleus through
the stria medullaris thalami. Efferents from the habenular nucleus pass to nuclei of the
midbrain by fasciculus retroflexus of Mynert, which project to the brain stem and
hypothalamus where it influences the autonomic centers in the pons and medulla including
the salivary nuclei.
Function. The habenular nucleus is concerned with integration of olfactory, visceral, and
somatic afferent pathways.

Pineal body is a conical projection attached by a stalk to the diencephalon. The superior
part of the stalk contains the habenular commissure; the inferior part of the stalk contains
the posterior commissure. The pineal gland possesses no nerve cells, but pinealocytes and
glia cells. Secretions of the pineal gland are melatonin and serotonin which are secreted
via the blood stream or via the CSF. The pineal secretions appear to influence other endo-
crine activities. The gland is most active during darkness.
Prof.Adel Kamel 77

SUBTHALAMUS

The subthalamus is situated between the thalamus and the tegmentum of the midbrain
dorsolateral to the hypothalamus. Ventrolaterally it is related to the internal capsule which
separates it from the globus pallidus. The subthalamic nucleus is located in the
ventrolateral part of subthalamus medial to internal capsule. Several important fibers
traverse the subthalamus in their way to the thalamus. These include 4 lemnisci,
dentatothalamic fibers and pallidothalamic fibers.. The principal connection of the
subthalamic nucleus is by two-way array with the globus pallidus through the subthalamic
fasciculus, and with the midbrain reticular formation.. The subthalamic nucleus is a site of
integration of motor control centers particularly by its connections with the basal ganglia
and midbrain reticular formation. It exerts regulatory influences upon the globus pallidus.

Fasciculus lenticularis arises from the globus pallidus traverses the internal capsule to
reach the prerubral field of Forel (fig.5-1). Ansa lenticularis arises from putamen and
globus pallidus and curves medially around the internal capsule to reach the prerubral field.
Fibers of fasciculus lenticularis and ansa lenticularis form the prerubral field of Forel in
the subthalamus. Some of these fibers synapse with the subthalamic nucleus and with the
nucleus of the prerubral field. The remaining fibers continue to reach the anterior ventral
nucleus of thalamus through thalamic fasciculus, and hypothalamus through
hypothalamic fasciculus. The thalamic fasciculus contains, as well, the contralateral
dentatothalamic and ipsilateral rubrothalamic tracts in their way to the thalamus.

Clinical notes

Lesion of one subthalamic nucleus releases the globus pallidus from the controlling effect
of the subthalamic nucleus. This is expressed by irregular forceful ballistic movements on
the opposite side (hemiballismus). Hemiballismus involves usually the proximal extremity
muscles and the limb involved is made to fly about in all directions. Hemiballismus can be
produced experimentally in monkey by destroying the nucleus. It is abolished by
destruction of the globus pallidus, or its outflow to the AVT or area 4.
Prof.Adel Kamel 78

Fig.5-1. Subthalamic nuclei and fibers.

METATHALAMUS

The metathalamus includes the medial (MGB) and the lateral geniculate (LGB) bodies.
The metathalamus is considered the thalamic center of hearing and vision.
Prof.Adel Kamel 79

THALAMUS

Shape and relations

The thalamus is egg-like mass of gray matter that forms the major part of the
diencephalon. They are situated on each side of the 3rd ventricle. Each thalamus has two
ends and four surfaces. The two ends are the anterior tubercle and the posterior end or
pulvinar. The 4 surfaces are:

a)Superior surface forms part of the floor of the stem of the lateral ventricle.

b)Medial surface forms part of the lateral wall of the 3rd ventricle. This surface is
connected to the opposite thalamus by interthalamic adhesions.

c)Lateral surface is closely related to the internal capsule.

d)Inferior surface lies on the subthalamus (in its middle part), and hypothalamus (in its
anterior part).

Internal Features

The gray matter of the thalamus is divided by a Y-shaped thin layer of nerve fibers, the
internal medullary lamina. The lamina is Y-shaped when viewed from above and
provides the basis for dividing the thalamus into three nuclear masses: anterior, medial
and lateral (fig.5-2). Each of these cellular complexes is further subdivided into a number
of named nuclei. The posterior part is a prominent swelling called the pulvinar which
overhangs the medial and the lateral geniculate bodies. Lateral to the thalamus lays the
external medullary lamina which consists of thalamocortical and corticothalamic fibers.
Between this and the internal capsule is located the reticular nucleus of the thalamus.

The medial nuclei include the small midline nucleus and the large dorsomedial nucleus.
Nuclei of the lateral group are divided into two groups; dorsal and ventral. The dorsal
group contains lateral dorsal and lateral posterior nuclei. The ventral group contains
anterior ventral, lateral ventral and posterior ventral nuclei. The posterior ventral group is
subdivided into three nuclei; medial, lateral and inferior (fig.5-2).
Prof.Adel Kamel 80

Overview of the thalamic nuclei and their connections

Each thalamic nucleus, after receiving impulses from subcortical pathways, relay these
impulses to designated areas of the ipsilateral cerebral cortex. Each cortical area in turn
projects back to the thalamic nucleus that project to it. These to-and-fro thalamocortical
and corticothalamic circuits allow the thalamus to modulate the motor, sensory and mental
functions of the CNS. In some cases precise point-to-point projection exist between
individual thalamic nuclei and restricted cortical zones with well defined sensory or motor
functions. Such thalamic nuclei are referred to as specific thalamic nuclei. All specific
nuclei lie within the ventral tier of the lateral nuclear group. Other thalamic nuclei receive
less distinct afferent input that does not have overt sensory or motor pathways. In turn
these nuclei connect with wider diffuse areas of the cortex including association and limbic
domains. These nuclei are referred to as non-specific nuclei. Non-specific nuclei include
nuclei of the dorsal tier, the whole anterior and medial nuclei.

Pathways received by the thalamus


The thalamus receives imput from (fig.5-3):
a) Sensory pathways (four lemnisci and optic tract).
b) Somatomotor pathways from basal ganglia and the cerebellum.
c) Reticular formation.
d) Limbic system.

Fig.5-2.Thalamic nuclei.
Prof.Adel Kamel 81

Fig.5-3.Projections of thalamic nuclei.


Prof.Adel Kamel 82

Functional nuclear groups of the thalamus

A)Specific nuclei (modality-specific nuclei)

1-Sensory relay nuclei -Posterior ventral nucleus -Medial

-Lateral

-Inferior

-Medial geniculate body

-Lateral geniculate body

2-Motor relay nuclei -Ventral anterior nucleus

-Ventral lateral nucleus

B)Non-specific nuclei

3-Limbic system relay nuclei -Anterior nucleus

-Dorsolateral nucleus

4-Association multimodal relay nuclei

-Dorsomedial nucleus

-Posterolateral nucleus

-Pulvinar

5-Other non-specific diffusely projecting nuclei

-Midline nucleus

-Centromedian nucleus

-Reticular nuclei
Prof.Adel Kamel 83

Thalamic projections

Thalamic projections mainly target the cerebral cortex. Thalamic nuclei connect with the
cortex that is most convenient to them and mirrors the location of the thalamic nucleus.
The olfactory tubercle, amygdale and the basal ganglia are the only subcortical structures
known to receive thalamic projections. Few if any thalamic efferents run to hypothalamus,
brain stem, cerebellum or spinal cord. Thalamic projections are among four peduncles;
anterior, superior, posterior and inferior.

Thalamic peduncles and their destinations


Thalamic peduncle Thalamic nucleus Destination
Anterior peduncle -Anterior nucleus -Cingulate gyrus.
Through anterior limb of -Dorsomedial N -Frontal cortex.
Internal capsule -Prefrontal cortex.

Superior peduncle -PLVNT -Postcentral gyrus.


Through posterior limb of -PMVNT (areas 3,1 and 2)
Internal capsule

Posterior peduncle -LGB -Visual areas.


Through retrolentiform part -Pulvinar
Of internal capsule
Inferior peduncle -MGB -Auditory areas.
Through sublentiform part
Of internal capsule
Prof.Adel Kamel 84

Connections of anatomically classified thalamic nuclei

ANTERIOR THALAMIC NUCLEAR COMPLEX

Anterior thalamic nucleus is part of the limbic system. It receives afferents from the
mammillary body via mammillothalamic tract and from the hippocampus through the
fornix. It reciprocally connects with the cingulate gyrus.

Function. Anterior nucleus is part of the Papez circuit involved in the emotion and recent
memory.

LATERAL THALAMIC NUCLEI

A-Dorsal Group

1-Lateral Dorsal Nucleus (LD)

It is an extension of the anterior nuclear complex. It receives input from the mammillary
body and hippocampus, and reciprocally connects to cingulate gyrus.

Function. LDN is part of the limbic system concerned with emotion and recent memory.

2-Lateral Posterior Nucleus (LP)

The lateral posterior nucleus receives vestibular impulses and relay them to posterior
insular region and parietal cortex. It is connected with the other thalamic nuclei and
reciprocally project to the parietal association areas (areas 5, 7).

Function. LPN projects information from thalamic nuclei to sensory association cortex.
Prof.Adel Kamel 85

B-Ventral Group

1-Ventral Anterior Nucleus (VA)

The VA receives afferents from the corpus striatum originating from the medial segment
of the globus pallidus and its homologoue, pars reticularis of substantia nigra. It is
reciprocally connected to motor areas of the frontal cortex particularly the motor,
premotor, supplementary motor and the frontal eye field (areas 4, 6 and 8).

Function. It plays an important role in the mechanism by which the basal ganglia
influence motor activity.

2-Ventral Lateral Nucleus (VL)

The ventral lateral nucleus has major input from the contralateral dentate nucleus of the
cerebellum and a minor input from ipsilateral basal ganglia, substantia nigra and red
nucleus. It has reciprocal connection with motor areas of the frontal lobe particularly the
primary motor area 4, premotor area 6 and area 8.

Function. VA plays a role in unconscious regulation of muscle tone. It influences the


motor mechanisms via the striatal motor system and the cerebellum.

3-Ventral Posterior Nuclei

Within the ventral posterior nucleus there is termination of all ascending pathways from
the spinal cord and brain stem that carry general sensory information from the
contralateral half of the body to a conscious level. Termination of these fibers in the ventral
posterior nucleus is highly organized somatotopically. The ventral posterior nucleus
projects to the primary sensory cortex in the postcentral gyrus, the upper 2/3 of the
sensory area receives from the PLVNT and the lower 1/3 receives from the PMVNT.
Prof.Adel Kamel 86

a) Ventral Posterolateral Nucleus (PLVNT)

The PLVNT is the extensive part of the ventral posterior nucleus. It receives sensory
modalites from the limbs and trunk via the spinal and medial lemnisci.

b) Ventral Posteromedial Nucleus (PMVNT)

The PMVNT is a smaller medial portion of the ventral posterior nucleus. It receives
sensory modalities from the face via the trigeminal lemniscus, and taste from the tongue
ascending from nucleus solitarius.

c) Ventral Posteroinferior Nucleus

It receives vestibular information from the vestibular nuclei via vestibulothalamic fibers
and projects to vestibular area of the somathetic cortex.

POSTERIOR THALAMIC NUCLEI

1-Pulvinar

The pulvinar is the largest thalamic nucleus. It is considered as an outgrowth of the lateral
posterior nucleus. It is connected with the other thalamic nuclei (especially the LGB and
MGB) and superior colliculus. Pulvinar projects to parietal and occiptotemporal
association cortex.

Function. Integration of somatic sensation with special sensations (especially vision and
hearing). Lesion in it leads to sensory aphasia.
Prof.Adel Kamel 87

2-Lateral Geniculate Body (LGB)

The LGB is part of the visual pathway. It is connected with the optic tract and projects to
the visual area through optic radiation.

3-Medial Geniculate Body (MGB)

The MGB is part of the auditory pathway. It is connected to the inferior colliculus, lateral
lemniscus and projects to auditory area.

MEDIAL THALAMIC NUCLEI

1-Dorsomedial Nucleus (DM)

DM nucleus receives subcortical and cortical connections.

Subcortical afferents originate from the hypothalamus, amygdale, substantia nigra, and
other thalamic nuclei including intralaminar nuclei and nuclei of the lateral group. DMN
has cortical (reciprocal) connections with the prefrontal cortex, orbitoinsular limbic cortex
and temporal lobe neocortex.

Function. DM Integrates sensory information (somatic, visceral and olfactory) and relates
these information to emotional feelings. It plays a role in control of mode and elation. It is
concerned with subjective feelings and personality of the individual. Bilateral lesion of the
DM results in loss of initiative, placidity and indifference to stimuli.

2-Midline Nucleus

The midline nucleus receives input from the reticular formation via the ascending
reticular activating system (ARAS) and from the globus pallidus. It projects reciprocally
and diffusely to the entire neocortex.

Function: Mediation of arousal response and modulation of consciousness.


Prof.Adel Kamel 88

OTHER THALAMIC NUCLEI

Intralaminar Nucleus is present in the Y-shaped internal medullary lamina. It includes


the centromedian nucleus (CM). Main connections of the CM is with the corpus
striatum, putamen and with other thalamic nuclei.

Reticular Nucleus covers the lateral part of the thalamus between the external medullary
lamina and the posterior limb of the internal capsule. It receives collaterals of both
thalamocortical and corticothalamic fibers. These nuclei are connected to the reticular
formation, other thalamic nuclei, spinothalamic, trigeminothalamic and the cerebral
cortex.

Function. The intralaminar and reticular nuclei are concerned with the mechanism by
which the cerebral cortex regulates the thalamic activity.

Blood supply of the thalamus


1-Posterior communicating artery gives the anterior thalamoperforating artery.
2-Posterior cerebral artery which gives the posterior choroid artery and the posterior
thalamoperforating artery.
3-Anterior choroid artery.
4-Thalamostriate of middle cerebral artery.

Functions of the thalamus


1-Integration of the sensory information converging to it. The thalamus appreciates
crude sensations. However the cerebral cortex is required for interpretation of sensations
based on past experiences. LGB& MGB of the thalamus constitute parts of the visual and
auditory pathways.
3-The dorsomedial nucleus lies on the pathway that is concerned with subjective feelings.
4-Midline and intralaminar nuclei influence the consciousness and alertness.
5-The thalamus plays a role in modulation of motor activity.
6-Multimodal nuclei are related to higher brain functions such as memory and learning.
7-Limbic nuclei play a role in emotion, behavior and recent memory.
Prof.Adel Kamel 89

Clinical Notes

Because the functions of the thalamus reflect the functions of the cerebrum, thalamic
lesions impair any of the three functions of the cerebrum: motor, sensory and mental.
1-The thalamus may undergo degeneration or be invaded by tumor or damaged by
hemorrhage. Damage of the posteroventral nuclei results in contralateral loss of sensations
and thalamic aphasia. Lesion of the right thalamus in a non-dominant hemisphere results
in that the patient gets lost and fails to recognize emotions of others.
2-Cutting thalamocortical radiations to the somatosensory cortex decreases the ability of
tactile localization and discrimination, and decreases position and movement appreciation,
but awareness of contact and vaguely localized pain and thermal sensations remain.
3-Sometimes with loss of sensation the patient, recovering from thalamic infarcts,
experiences severe spontaneous pain in the opposite side of the body. This thalamic
syndrome is called Dejerine-Roussy syndrome (anesthesia dolorosa; painful anesthesia).
4-Alterations in emotions are produced when the pathways between the dorsomedial
nucleus and the frontal association cortex are severed, e.g. in frontal lobectomy. Patients
suffer from emotional instability, loss of orientation to place, time and persons.
5-Bilateral thalamic lesion may lead to loss of consciousness and indifference to stimuli.
Prof.Adel Kamel 90

Summary of the thalamus

Sensory modalities converge on the thalamus and integrate with each other and
diverge to many destinations. The thalamus is involved in activities of all major regions
of the CNS; the sensory system, most of the cerebral cortex, corpus striatum, cerebellum,
hypothalamus, subthalamus and brain stem RF.
Thalamic nuclei are either specific or non specific according to their areas of the cortex
they project to. Specific-nuclei lie within the ventral part of the lateral nuclei. They project
to specific sensory and motor areas of the cortex. Non-specific nuclei project to diffuse
cortical areas.
Sensory modalities are received into the sensory relay nuclei. These specific nuclei are the
PLVNT, PMVNT, MGB and LGB. PLVNT receives spinal and medial lemnisci, and
project to primary sensory areas. MGB receives lateral lemniscus and projects to auditory
areas. LGB receives optic tract and projects to visual areas.
Connection of the dorsomedial nucleus with the prefrontal cortex and hypothalamus is
regarded as a center for integration of visceral and somatic functions. By hypothalamic
connections it is involved in autonomic and endocrine activities. Also DMN plays an
important role in emotional content, subjective feelings and consciousness of "self".
The anterior thalamic nuclei integrate a complex input along the mammillothalamic tract,
hypothalamus, limbic system and cingulate gyrus. It has reciprocal connections with all
these sources. Such arrangements are concerned in complex mechanisms of recent
memory and the balance between repetitive and stereotyped forms of behavior.
Anterior ventral nucleus and lateral ventral nucleus are sites of interactions of outflows
from the corpus striatum and cerebellum with other thalamic nuclei, and projecting to
motor and premotor areas. They play an essential role in full locomotor control.
The non-specific nuclear group has connections with each other and with the specific-
thalamic nuclei, receiving from many sensory paths, reticular formation, cerebellum,
corpus striatum and via cortical projection they influence the arousal reaction.
Prof.Adel Kamel 91

HYPOTHALAMUS

Definition

The hypothalami are the most ventral parts of the diencephalons that are responsible for
internal homeostasis. The hypothalamus integrates interoceptive signals from internal
organs and makes appropriate adjustments to internal environment through its input and
output systems. The hypothalamus extends from the region of the optic chiasma to the
mammillary bodies. It lies below the thalamus and forms the floor and inferior part of the
lateral wall of the third ventricle.

Relations (fig.5-4)

1-Anteriorly -Optic chiama.

-Lamina terminalis.

-Anterior commissure.

2-Posteriorly -Tegmentum of the midbrain.

3-Laterally -Internal capsule.

4-Inferiorly -Optic chiasma.

-Tuber cinerum.

-Infundibulum.

-Mammillary bodies.

5-Superiorly -Hypothalamic sulcus.


Prof.Adel Kamel 92

Fig.5-4.Relations and nuclei of the hypothalamus.


Prof.Adel Kamel 93

Hypothalamic nuclei (fig.5-5)

In a cephalocaudal direction 3 hypothalamic nuclear regions are recognized:

_________________________________________________________________

(1)Supraoptic Area (2)Tuberal Area (3)Mammillary Area

(Anterior group) (Middle group) (Posterior group)

________________________________________________________________

-Supraoptic nucleus -Dorsomedial nucleus -Posterior nucleus

(ADH) (Satiety center) (sympathetic stim.)

-Paraventricular nucleus -Ventromedial nucleus -Mammillary nucleus

(Oxytocin) (Control food intake) (Papez circuit of emotion)

-Anterior nucleus -Lateral nucleus

(Parasympathetic stim.) (Hunger center)

-Suprachiasmatic nucleus -Tuberal nucleus

(Retinal input, control (Releasing factors)

Circadian rhythm) -Arcuate nucleus -

-Preoptic nucleus (Releasing factors)

(Control release of

gonadotrophins)
Prof.Adel Kamel 94

Fig.5-5.Hypothalamic nuclei; functions and dysfunctions.


Prof.Adel Kamel 95

Hypothalamic connections

The hypothalamus which lies in the center of the limbic system has many extensive and
complex connections. Some fibers are organized into definite bundles, while others are
diffuse and difficult to trace. Connections are neural or in the form of circulatory signals.

NEURAL

Part of the CNS Afferents Efferents

Cerebral cortex -Frontoorbital cortex (MLB) -Diffuse cortical


-Hippocampus (fornix)

Subcortex -Amygdala (stria terminalis)

Diencephalon -Retina
-DMN of thalamus -AN of thalamus
-HH tract and portal system
Brainstem
Midbrain -RF &periaqueductal gray mater -mammilotegmental T
Pons -Raphi nuclei -RF
-N of locus ceruleus
Medulla -N solitaries -DMN of vagus

Spinal cord -Ascending sensations -preganglionic fibers

CIRCULATORY SIGNALS
-Physical signals (temperature and osmolality) -Releasing factors
-Chemical signals (blood glucose and acid-base) -Oxytocin
-Hormonal signals (feed back) -ADH
Prof.Adel Kamel 96

Afferents to hypothalamus
1-Visceral and somatic afferents reach the hypothalamus through collaterals of the
lemnisci and the reticular formation.
2-From the olfactory region reach the lateral nuclei of the hypothalamus through the
medial forebrain bundle (MFB). MFB is formed mainly of unmyelinated fibers and
represents pathway for visceral and autonomic system.
3-From the frontal and orbital cortex project directly to the hypothalamus.
4-From the dorsomedial and midline nuclei of the thalamus to the hypothalamus.
5-From hippocampus pass through the fornix to the medial mammillary nucleus.
6-From the amygdaloid nucleus to the hypothalamus (preoptic area and anterior nucleus)
through the stria terminalis.
7-From tegmentum of the midbrain to the lateral mammillary nucleus.
8-From raphe nuclei of the brain stem (seretonergic fibers), locus ceruleus (noradrenergic
fibers) and septal nuclei.

Efferents from hypothalamus

1-Mammillothalamic tract originates from the mammillary body to the anterior nucleus of
thalamus. This pathway is relayed in the cingulate gyrus, and by this the hypothalamus
controls and is brought under the control of cingulate gyrus.
2-Mammillotegmental tract originates from mammillary body to the reticular formation.
3-Descending fibers in the form of dorsal longitudinal bundle to the brain stem and spinal
cord (hypothalamospinal tract), influence neurons of autonomic nervous system.
4-Hypothalamohypophyseal tract descends from supraoptic and paraventricular nuclei to
posterior lobe of pituitary gland. Hormones pass along the axons and are released at axon
terminals. Antidiuretic hormone is produced by the supraoptic nucleus and oxytocin is
produced by the paraventricular nucleus. The hypothalamus controls the anterior pituitary
via humeral portal system by which releasing hormones and release-inhibiting hormones
are produced in the neurons of the arcuate nucleus to the secretory cells of the anterior
pituitary.
Prof.Adel Kamel 97

Functions of the hypothalamus

1-High autonomic control: The anterior hypothalamic area and the preoptic area
stimulate the parasympathetic responses; the posterior and lateral hypothalamic nuclei
stimulate the sympathetic responses.
2-Endocrine control: The arcuate and tuberal nuclei control the release of trophic
hormones of the anterior pituitary. Preoptic nucleus controls release of gonadotrophins.
3-Regulation of temperature: The anterior nuclei control heat loss and the posterior
nuclei control heat gain. Lesion of anterior nucleus produces hyperthermia.
4-Regulation of food and water intake: Hunger center is situated in the lateral nuclei
while satiety center is situated in the medial nuclei. Lesion of the lateral nuclei results in
starvation. Lesion of the medial nuclei results in obesity.
5-Emotion and behavior: Hypoyhalamus is the integrator of afferent information received
from other areas of the nervous system and brings about the physical expression of emotion
(increase heart rate, dryness of mouth, flushing or pallor and sweating).
6-Control of circadian rhythm: Sleeping and wakefulness are controlled by
hypothalamus. Lesion of anterior part of hypothalamus interferes with the rhythm of
sleeping and waking. Suprachiasmatic nucleus, which receives afferents from the retina,
plays a role in controlling the biological rhythms.

Clinical Notes
Hypothalamus may be the site of inflammation, neoplasm or vascular injury.
Hypothalamic lesion may produce one or more of the following manifistations:
1-Obesity with genital hypoplasia.
2-Sexual disorders in children results in sexual retardation or precocity. In adults sexual
disorders results in impotance or amenorrhea.
3-Hyperthermia or hypothermia.
4- lesion in supraoptic nuclei results in diabetes insipidus.
5-Disturbances of sleep.
6-Emotional disorders in the form of attacks of unexplained weeping, laughter, depressive
reactions or mania.
Prof.Adel Kamel 98

STUDY QUESTIONS ON CHAPTER 5


Choose the correct answer
1-Stria medullaris thalami connects 6-Hypoth. N. secretes releasing
factors
a)Habenular nucleus to thalamus. a)Arcuate.
b)Habenular nucleus to amygdala. b)Mamillary.
c)Thalamus to hypothalamus. c)Suprachiasmatic.
d)Thalamus to pineal body. d)Posterior.

2-Prerubral field is found in 7-FALSE about medial


forebrain bundle
a)Subthalamus.
b)Midbrin. a)Runs medial to internal capsule.
c)Thalamus. b)Consists mainly of myelinated fibers
d)Hypothalamus. c)Pathway for visceral system.

3-Trimodal association thalamic nucleus is d)Connect olfactory area with hypoth.


a)Pulvinar.
b)Anterior nucleus.
c)PLVNT. 8-Nucleus of hypothalamus whose
d)MGB. lesion leads to diabetes insipidus

4-Structure designating thalamic nuclei is a)Paraventricular.


a)Internal medullary lamina. b)Supraoptic.
b)Reticular nucleus. c)Preoptic.
c)Stria medullaris thalami. d)Anterior.
d)Medial forebrain bundle. 9-False about mamillary body

5-Which thalamic nucleus connects with a)Projects to LV thalamic nucleus.


Limbic pathway b)Receives input from hippocampus.
a)Pulvinar. c)Receives input from raphe nuclei.
b)Anterior nucleus. d)Located behind tuber cinerum.
c)Centromedian nucleus. 10-INCORRECT about LGB
d)Lateral ventral nucleus. a)Receives input from optic tract.
b)Supplied by anterior choroid artery.
c)Receives input from lateral lemniscus.
d)Projects to lingual gyrus.
Prof.Adel Kamel 99

CHAPTER 6

BASAL GANGLIA
Objectives
-Definition, description and classification of basal nuclei.
-Main connections of basal ganglia.
-Role of basal nuclei in motor control.
-Anatomical basis for basal ganglia lesions.

Definition of basal ganglia (nuclei)

The term basal ganglia is applied to a collection of masses of gray matter situated within
each cerebral hemisphere in close relation to the diencephalon but separated from it by the
internal capsule. Basal ganglia represent subcortical nuclei derived from telencephalon.

Clasification of basal nuclei

Caudate nucleus Neostriatum


Corpus striatum Lentiform nucleus

Basal Claustrum Putamen

Ganglia Globus pallidus Paleostriatum

Amygdaloid Archistriatum
Prof.Adel Kamel 100

Phylogenetically the basal ganglia are classified into very old amygdaloid complex or
archistriatum, old globus pallidus or paleostriatum and new caudate nucleus and putamen
or neostriatum. The caudate and putamen are functionally similar and commonly referred
to as the striatum.

The neostriatum and the paleostriatum together form corpus striatum.

AMYGDALOID NUCLEUS

The amygdaloid nucleus is considered part of the limbic system and recently excluded
from the basal ganglia. It is situated in the temporal lobe close to the uncus of the
parahippocampal gyrus. It lies in front and partly above the tip of the inferior horn of the
lateral ventricle. It is fused with the tip of the tail of caudate nucleus. The stria terminalis
emerges from its posterior aspect, and runs as a bundle of nerve fibers posteriorly in the
roof of the inferior horn of the lateral ventricle medial to the tail of caudate nucleus. The
stria terminalis follows the curve of the caudate nucleus and comes to lie in the floor of the
body of the lateral ventricle where it occupies the groove between the caudate nucleus and
the thalamus. The stria terminalis end in the lateral and preoptic hypothalamic nuclei.
Amygdaloid nucleus receives afferents from the lateral olfactory tract, reticular formation,
septal nuclei, and limbic and sensory cortex. It projects efferents to dorsomedial nucleus of
thalamus, hypothalamus (via stria terminalis), habenular nuclei (via stria medullaris
thalami), and sensory and limbic association cortices.

Function of amygdaloid nucleus


The amygdaloid nucleus is considered part of the limbic system which produces activities
associated with feeding and nutrition. It regulates visceral activity and emotional
behavior. Its stimulation may cause aggressive behavior.
Destruction of the amygdaloid nucleus results in Kluver-Bucy syndrome; characterized
by psychic blindness (visual agnosia), loss of fear and anger, increase appetite and sexual
activity, and decrease aggressiveness and decrease emotional instability. There is no
disturbance in memory. Precise stereotactic lesion of the amygdaloid nucleus in man
reduces emotional excitability and brings about normalization of behavior in patients with
severe emotional disturbances.

Claustrum
Claustrum is a thin plate of gray matter lying between the lentiform nucleus and the insula.
It is separated from these structures by two white laminae; the external capsule medially
and the extreme capsule laterally. Some consider the claustrum as part of the striatum, but
there is evidence that it originated from the deep layers of the insular complex from which
it splits off.
The functions and connections of the claustrum are unknown.
Prof.Adel Kamel 101

CAUDATE NUCLEUS

The caudate nucleus is C-shaped mass of gray matter that is closely related to the lateral
ventricle and lies lateral to the thalamus. It is formed of head, body and tail. The head is
large, rounded and forms the lateral wall of the anterior horn of the lateral ventricle. The
head is continuous inferiorly with the putamen. The body of caudate is long and narrow
and forms part of the floor of the body of the lateral ventricle. Here it is separated from the
thalamus by a groove containing stria terminalis and the thalamostriate vein. The tail of
caudate is long and slender and on reaching the caudal limit of the thalamus, it arches
ventrally and runs forward in the roof of the inferior horn of the lateral ventricle. The tail
terminates anteriorly in the amygdaloid nucleus.

LENTIFORM NUCLEUS

The lentiform nucleus is a wedge shaped mass of gray matter, whose convex base is
directed laterally and its blade medially. The two other surfaces converge to a medial apex
which lies against the genu of the internal capsule. It is related medially to the internal
capsule which separates it from the caudate nucleus and the thalamus. Laterally, it is
related to a thin sheet of white matters the external capsule that separates it from the
claustrum. The claustrum separates the external capsule from the subcortical white matter
of the insula. A vertical plate of white matter divides the lentiform nucleus into a large
darker lateral portion, the PUTAMEN and an inner lighter portion, the GLOBUS
PALLIDUS. At its anterior end, the putamen is continuous with the head of caudate
nucleus. This connection is termed nucleus accumbens septalis which has connections
with the limbic system. The globus pallidus consists of two divisions; lateral and medial
segments separated by a thin sheet of fibers, the medial medullary lamina. The medial
(smaller) segment shares many similarities in cells and connections with the pars reticularis
of the substantia nigra. The two are regarded a single entity in function.
Prof.Adel Kamel 102

Connections of the corpus striatum (fig.6-1)

Afferents

1-Corticostriate fibers are projected from the greater part of the cortex, particularily the
sensorimotor cortex to the caudate nucleus and the putamen. Fibers project bilaterally
crossing the midline through the corpus callosum and enter the caudate nucleus via the
subcallosal fasciculus, and the putamen via the external capsule. More anterior regions of
the frontal lobe and other association cortices project mainly to the caudate nucleus while
motor regions project mainly to the putamen. For this, the putamen is considered the
motor part of the straitum and the caudate having the most associative functions.

2-Thalamostriate fibers arise from the centromedian nucleus of the thalamus. Fibers
traverse the internal capsule and enter putamen.

3-Nigrostriate fibers exert modulatory (mostly inhibitory) effect on the striatal neurons.
Dopamine regulates the releases of other neurotransmitters from axon terminals.
Dopaminergic neurons are rich in melanin pigments.

Efferents

1-Striopallidal fibers pass from the caudate nucleus ventrally via the internal capsule to
reach putamen, and from the putamen fibers project medially to the globus pallidus.

2-Strionigral fibers project mainly from the caudate nucleus to substantia nigra.

It should be noted that the two output nuclei of the basal ganglia are the globus pallidus
and the substantia nigra. These outputs use GABA as a neurotransmitter to inhibit their
targets. The main input to these nuclei is from the striatum and the subthalamic nucleus.
Prof.Adel Kamel 103

Connections of the globus pallidus

Afferents

1-Striopallidal fibers come from putamen and caudate nucleus.

2-Nigropallidal fibers project from substantia nigra to globus pallidus. Some fibers reach
the caudate nucleus and the remaining fibers continue to the putamen and globus pallidus.

3-Subthalamopallidal fibers project from the subthalamic nucleus to globus pallidus


through the subthalamic fasciculus.

Efferents

1-Pallidothalamic fibers: Fasciculus lenticularis and ansa lenticularis reach the prerebral
field of Forel in the subthalamus. Some of the fibers synapse with subthalamic nucleus and
nucleus of the prerubral field. It is noticed that the lenticular fasciculus traverses the
internal capsule whereas the ansa lenticularis sweep around the posterior limb of the
internal capsule forming a loop (ansa) to reach the prerubral field. The remaining fibers
reach ventral anterior and ventral lateral nuclei of the thalamus through the thalamic
fasciculus. Pallidothalamic fibers constitute the main outflow from the basal ganglia. VA
and VL thalamic nuclei project excitatory fibers to the motor cortex (primary motor and
supplementary motor cortex).

2-Pallidotegmental fibers descend to terminate in brain stem tegmentum in the


pedunculopontine nucleus which lies at the boundary between midbrain and pons. This
region has been termed the mesencephalic locomotor region in lower mammals, since it is
involved in the regulation of quadrupedal progression.

3-Other efferents of the globus pallidus include; a)Subthalamic nucleus, b)Red nucleus,
c)Midbrain RF, d)Epithalamus and Hypothalamus, and e)Inferior olivary nucleus.

Globus pallidus project to midbrain reticular formation. This is a part of the brain stem
reticular formation and so is connected with the cells that give rise to the reticulspinal tract.
This represents a more direct route than through the thalamus for the basal ganglia output
to reach the lower motor neurons. Limbic component of basal ganglia is implicated in
emotional behavior.
Prof.Adel Kamel 104

Fig.6-1. Connections of the basal ganglia.


Prof.Adel Kamel 105

Functions of corpus striatum

1-Definition and function of the extrapyramidal system:


The extrapyramidal system consists of all motor pathways of the brain that ultimately
influence LMNs but do not send their axons directly into the pyramidal tract. This includes
the circuitry of the basal ganglia, and the reticulospinal, rubrospinal, vestibulospinal,
olivospinal and, tectospinal tracts. Signs of pyramidal diseases differ from the causes and
signs of extrapyramidal diseases.
2-The phylogenetic series illustrate a gradual increase in the dependence of volitional
movement on the pyramidal tract. Volitional motor activity is extrapyramidal in
submammalia, mixed pyramidal/extrapyramidal in subhuman mammalia and essentially
pyramidal in human.
-In man the corpus striatum maintains a postural background for voluntary activities,
reinforcing and steadying movements and postures of cortical origin, but is incapable of
initiating such movements. The cerebral cortex projects to lower motor neurons through
two main channels; (a)long fibers in the pyramidal tract concerned with fine isolated
skilled movements, and (b)short fibers passing to extrapyramidal nuclei concerned with
mechanisms for postural adjustments and gross movement patterns that are largely
reflex in character.
-Recent concepts of the role of the basal ganglia consider their function to facilitate
movements that are appropriate and to inhibit unwanted movements that are
inappropriate. When a movement is initiated, the cerebral cortex discharges ecitatory
impulses to neostriatum. The striatum has two routes by which it controls activity of the
globus pallidus; direct and indirect. The direct pathway through which striatopallidal will
directly inhibit the medial segment of the globus pallidus. Since medial globus pallidus
output neurons are themselves inhibitory, this leads to disinhibition of the motor thalamus.
The increased activity of the thalamic neurons causes excitation of the cerebral cortex. The
effect of activation of the direct pathway is, therefore, to facilitate ongoing movements.
The indirect pathway involves the subthalamic nucleus. Efferents from the striatum
terminate in the lateral segment of the globus pallidus inducing inhibition. The main
projection of the lateral pallidum is to the subthalamic nucleus which becomes
disinhibited. Activation of the subthalamic nucleus causes activation of the medial segment
of the globus pallidus and, in turn, inhibition of the motor thalamus and cortex. The result
is inhibition of the unwanted movements.
3-Corpus striatum plays a role in mentation (lesion of caudate results in impairment of
cognitive functions), emotion and behaviour (lesion of nucleus accumbens impairs
affective behaviour), planning and sequencing skills (lesion of pallidum produces
depression, apathy and reduces planning and skills).
Prof.Adel Kamel 106

Clinical Notes

1-Diseases of the extrapyramidal system are associated with dyskinesia (abnormal


involuntary movements) and disturbances of muscle tone. Positive disturbances (tremors,
athetosis, chorea and ballism) are believed to be the result of release phenomena. For
example the subthalamic nucleus normally exerts regulating influences upon the globus
pallidus. A lesion in the subthalamic nucleus releases the globus pallidus from this
controlling influence which is expressed by bursts of irregular forcefull ballistic
movements. In Parkinsonism, lack of dopamine (dopamine stimulates medial pallidum and
inhibits lateral pallidum) causes underactivity of the direct pathway and overactivity of the
indirect pathway. These changes exacerbate the abnormal overactivity of the medial
pallidum inducing dyskinesia.
2-Rigidity or increased muscle tone is a manifestation of extra-pyramidal lesion. Rigidity is
present in antagonistic muscle groups i.e. in both extensor and flexor muscles. Rigidity is
cog-wheel or clasp knife (passive movements are interrupted by jerks).
3-A unifying feature of its lesion is absence of muscle paralysis and absence of sensory
loss.
4-Depression, apathy, impairement of cognitive functions an disturbed affective behavior
are common manifestations of basal ganglia lesions.
Prof.Adel Kamel 107

SUMMARY OF THE BASAL GANGLIA


The basal ganglia are masses of gray matter situated within the cerebral hemisphere and
are composed of: 1-Caudate nucleus, 2-Lentiform nucleus: (a) Putamen and (b)
Globus pallidus and 3-Amygdaloid complex.
The amygdaloid complex is part of the limbic system. It receives afferents from
olfactory bulb and the temporal association cortex. Its main efferent is to the hypothalamus
(through stria terminalis). The amygdaloid nucleus is concerned with regulation of visceral
activity and emotional behavior. Destruction of amygdaloid nucleus produces psychic
blindness, loss of fear and anger, increase in appetite and sexual activity, and decrease
emotional instability and aggression.
The caudate nucleus is a C.-shaped mass of gray matter that is closely related to the
lateral ventricle. It is formed of head, body and tail.
The lentiform nucleus is a wedge-shaped mass of gray matter which is related medially
to the internal capsule and laterally to the external capsule. A vertical plate of white
matter divides the nucleus into a large lateral portion, putamen, and an inner lighter
portion, globus pallidus. Caudate nucleus and putamen together are considered the striatum
which receives afferents from the cerebral cortex, thalamus and substantia nigra. Striatum
projects to globus pallidus and substantia nigra. Globus pallidus receives afferents from
corpus striatum, substantia nigra and subthalamic nucleus. Pallidal efferents enter
the ventral anterior nucleus of thalamus.
Basal ganglia in man are utilized for automatic movements concerned with postural
adjustments but they are incapable of initiating such movements. Basal ganglia play a
role in selection of movements, mentation, emotion and behaviour.
Lesion in the basal ganglia may produce abnormal involuntary movements as well as
disturbances of muscle tone. There are accompanying depression, apathy and
impairment of skills.
Prof.Adel Kamel 108

CHAPTER 7

LIMBIC SYSTEM
Objectives
-Concept of limbic lobe and limbic system.
-Circuits of limbic system (intrinsic, emotion and behavior)
-Functions and dysfunctions of the lmibic system.

Definition
The limbic system is the system underlying emotional and behavioral expression. It plays
a role in feeling, feeding, fighting, fleeing, space mapping, and mating activity. The limbic
system expresses itself through the hypothalamus via the autonomic nervous system.

The limbic lobe is a large arcuate convolution situated on the medial surface of the
hemisphere surrounding the upper part of the brain stem. It is formed of cingulate gyrus
and parahippocampus.

The term limbic system (fig.7-1) is more extensive term. It includes the limbic lobe as well
as associated subcortical nuclei such as; amygdaloid complex, septal nuclei (these are
nuclei within the midline close to the anterior commissure and considered pleasure centers
of the brain), hypothalamus, anterior thalamic nuclei, hippocampal formation (formed
of; a)dentate gyrus, b)hippocampus proper, and c)subiculum). Dentate gyrus and
hippocampus are primitive trilaminar cortex (archicortex), whereas the subiculum has
graded cortical layers from 4 to 6 layers where it merges with the neocortex.

Major limbic fiber systems

The alveus, the fimbria, the fornix, the mamillothalamic tract, the stria terminalis,
diagonal band of Broca and cingulum constitute the connecting pathways of the limbic
system. The entorhinal and subicular cortices are the primary areas responsible for
extrinsic connections of the hippocampal formation.

.
Prof.Adel Kamel 109

Fig.7-1.Components of the limbic system.


Prof.Adel Kamel 110

Hippocampal formation

The Hippocampus (=sea horse) is a curved elevation of gray matter that lies in the floor of
the inferior horn of the lateral ventricle. Its anterior end is expanded to form pes
hippocampus. Its ventricular surface is covered by a thin layer of white matter called the
alveus. The alveus consists of nerve fibers that originated in the hippocampus and
converge medially to form a bundle called the fimbria. The fimbria becomes continuous
with crus of the fornix. Column of the fornix carries efferents to:

a) Posterior to anterior commissure projecting to mammillary body.

b) Anterior to anterior commissure projecting to septal nuclei.

c) Through stria medullaris thalami to habenular nuclei.

The Dentate gyrus is a notched gray matter that lies between the fimbria of the
hippocampus and the parahippocampal gyrus. Posteriorly, the dentate gyrus becomes
continuous with the indusium griseum. The indusium griseum is a thin vestigial layer of
gray matter that covers the superior surface of the corpus callosum, extending around the
genu of corpus callosum and descends into the septal area.

-The Subiculum is the cortex from the entorhinal area to the hippocampus with gradual
change in its structure from a modified six-layered to four layered cortex.

Connections (circuits) of the limbic system

A) Intrinsic circuit

Intrinsic connections of the hippocampal formation are called the trisynaptic pathway
(fig.7-1). It begins with entorhinal cortex projections to dentate gyrus granule cells.
Granule cells project to pyramidal cells in the hippocampus. Pyramidal cells project to the
subiculum. The subiculum projects back to the entorhinal cortex. Intrinscic circuit of the
hippocampal formation is concerned with processing of information. Several feed back
loops modify the flow of information in the hippocampal formation.
Prof.Adel Kamel 111

Hippocampus

Dentate Gyrus Subiculum

Entorhinal cortex

Intrinsic circuit of limbic system

B) Papez circuit of emotion

CINGULATE GYRUS

Anterior thalamic peduncle Cingulum

THALAMUS PARAHIPPOCAMPUS

(Anterior nucleus)

Mamillothalamic tract Perforant pathway

HIPPOCAMPUS

Fornix

HYPOTHALAMUS

(Mamillary body)
Prof.Adel Kamel 112

Papez suggested that certain rhinencephalic and limbic pathways provided the anatomical
basis for emotions and their expression through visceral and instinctual actions such as
those involved in feeding, mating and aggression. Papez circuit consists of feed-in/feed-out
pathways between cortical and subcortical centers. Cingulate gyrus connects with the
parahippocampal gyrus and piriform area of the temporal lobe via the cingulum. Cingulate
gyrus is considered a center of feeling good. Entorhinal cortex (anterior part of
parahippocampus) connects with the hippocampus via the temporoammonic tract
(perforant pathway). Hippocampus connects with the mammillary body via the fornix.
Mammillary body connects with the anterior nucleus of the thalamus via the
mammillothalamic tract. The anterior nucleus of the thalamus connects with the cingulate
gyrus via the anterior thalamic peduncle completing the circuit. Amygdala connects with
this circuit through the ansa lenticularis.

C) Circuit of behaviour

Sensory information received from the outside reach primary sensory areas (somathestic,
visual, auditory…) and refined in the parieto-occipital association areas (perceptuospatial
function). This information is then conveyed to the frontal association areas involved in
planned behaviour (regulation of behaviour) and also to the inferior temporal association
areas (fig.7-2), where information can reach a supramodal status and meaning (semantic
processing). Entry of information into the limbic system is either directly to the amygdala
or indirectly to the hippocampal formation via the entorhinal area. The amygdala provides
an affective connotation to experience and especially that relevant to social stimuli.
Affect is a primitive feelings derived from the sensory autonomic input from body organs
to the hypothalamus. The flow of information into the hippocampal formation permits a
link to previous experience since the hippocampal formation is essential for learni ng and
memory. The limbic system is able to influence motor responses appropriate to its
informational analysis through projection to nucleus accumbens (ventral caudate nucleus).
The powerful input to the limbic system from the neocortical association areas link
complex (goal directed) behaviour to more primitive instinctive behaviour.
Prof.Adel Kamel 113

Fig.7-2.Interconnections between cortex and limbic system.

Functions of the limbic system

The limbic cortical areas are highly interconnected and have prominent connections with
the limbic subcortical (diencephalic) areas. The interrelated areas in the limbic cortex and
diencephalon form the limbic system serving emotional and motivational behavior as
well as learning and memory. The anterior portion of the limbic system (amygdaloid
nucleus and cingulate gyrus) regulates affective behavior and its visceromotor response.
The posterior portion of the system (hippocampal formation) is mainly involved in the
temporary storage of information (learning and recent memory). It is to be noted that the
emotional state can influence learning and memory, and elements of learning and memory
(habituation and orientation) can alter the emotional status.
The limbic system is involved in the transfer of declarative memory (memory of facts)
from short-term to long term memory. The limbic lobe is rostrally connected to the
olfactory bulb; this fact suggested that there is relation of the limbic lobe to olfaction,
hence the name rhiencephalon.
The limbic system is at the interface between somatic and visceral areas of the brain and
could relate these systems to each other, and to the behavior of the individual.

Clinical Notes
1-Disorders of the limbic cortex have been linked to psychiatric diseases, e.g.
schizophrenia, autism and amnesia. Pyramidal neurons of the hippocampus are abnormally
oriented in schizophrenic patients.
2-Damage to the hippocampal formation leads to memory and learning deficits.
3-Abnormal pyramidal neurons in the hippocampus may initiate epilepsy.
4-Patients with Alzhemer's disease show extensive damage in the hippocampal formation
resulting in memory impairment.
Prof.Adel Kamel 114

STUDY QUESTIONS ON CHAPTER 6 and 7

Choose the correct answer

1-Which thalamic nucleus project 6-A structure which is exclused from


To the striatim basal ganglia to limbic system is
a)Centromedian. a)Amygdaloid nucleus.
bDorsomedial. b)Putamen.
c)Ventrolateral. c)Nucleus accumbens.
d)PLVNT. d)Caudate nucleus/

2-Globus pallidus projects to thalamus via 7-FALSE about hippocampal


formation
)a)Stria medullaris. a)Major input via entorhinal cortex.
b)Stria terminalis. b)Major output is via the fornix.
c)Ansa lenticularis. c)Connects to thalamus via internal
capsule.
d)Fasciculus retroflexus. d)Includes dentate gyrus and subiculum.

3-Is not afferent fibers of neostriatum 8-Perforant pathway connects


a)Corticostriate. a)Thalamus and hypothalamus.
b)Nigostriate. b)Hippocampus and parahippocampus.
c)Subthalamostriate. c)Thalamus and cingulated gyrus.
d)Thalamostriate. d)Dentate gyrus to Ammon's horn.

4-Is not a sign of Parkinsonism 9-Center of pleasure is in


a)Rigidity. a)Hippocampus.
b)Resting tremors. b)Septal nuclei.
c)Sensory impairement. c)Dorsomedial nucleus of thalamus.
d)Mask-like face. d)Lateral nucleus of hypothalamus.

5-A coronal section through the genu of 10-Is not a function of limbic system
Internal capsule would bisect a)Space mapping.
a)Caudate nucleus. b)Recent memory.
b)Globus pallidus. c)Affective behavior.
c)Thalamus. d)Stereotyped movements.
d)Subthalamus.
Prof.Adel Kamel 115

COMPREHENSIVE QUESTIONS

SAQs

1-Describe order neurons of pain pathway from the right side of the body and face.
2-Name components of the pyramidal tract and state the order of their descent.
3-Name facial nerve nuclei according to their functional classification.
4-Describe the cerebellar connections based on its functional three lobes.
5-State the connections of the specific thalamic nuclei.
6-State connections of the globus pallidus and explain how it controls motor activity.
7-Name the main circuits of the limbic system and their significances.

MCQs
1-The ability to recognize an object in the hand depends on integrity of which tract
a)Dorsal spinocerebellar.
b)Cuneate.
c)Spinpolivary.
d)Anterior spinothalamic.

2-Pyramidal tract undergoes decussations at


a)Internal capsule.
b)Rostral pons.
c)Caudal medulla.
d)Crus cerebri.

3-Corticospinal tract receives contribution from all the following EXCEPT


a)Prefrontal cortex.
b)Somathetic area.
c)Paracentral lobule.
d)Premotor cortex.

4-The tract whose lesion results in contralateral leg dystaxia is


a)Dorsal spinocerebellar.
b)Gracile.
c)Ventral spinocerebellar.
d)Spinoolivary.

5-Which of the following nuclei does not project to the cerebellum


a)Lateral reticular.
b)Inferior olivary.
c)Nucleus ambiguous.
d)Accessory cuneate.

6-All the following statements concerning the trapezoid body are correct EXCEPT
a)It is found in the ventral pontine tegmentum.
b)It contains auditory fibers.
c)It contains the medial longitudinal bundle.
d)It projects to the lateral lemniscus.
Prof.Adel Kamel 116

7-All the following statements concerning olfactory pathway are correct EXCEPT
a)Primary olfactory cortex is present in the parahippocampus.
b)Olfactory fibers are special visceral afferents.
c)Olfactory nerve fibers synapse with mitral cells.
d)Olfaction is the sensation which bypass the thalamus to reach the cortex.

8-Signs of cerebellar dysfunction include all the followings EXCEPT


a)Hypotonia.
b)Slurred speech.
c)Resting tremors.
d)Unstable gait.

9-The following statements about dorsomedial nucleus of thalamus are correct


EXCEPT
a)It receives input from amygdala.
b)It is part of the limbic system.
c)It is part of extrapyramidal system.
d)It has reciprocal connections with the prefrontal cortex.

10-All are manifestations of Horner syndrome EXCEPT


a)Drooping of the eyelid.
b)Pubillary constriction.
c)Dryness of skin.
d)Dizziness.

11-The fornix connects the mamillary body with


a)Hippocampus.
b)Anterior nucleus of thalamus.
c)Cingulate gyrus.
d)Amygdala.

12-Striatum receives input from all the followings EXCEPT


a)Substantia nigra.
b)Subthalamic nucleus.
c)Centromedian nucleus.
d)Motor cortex.

13-All of the flowing statements concerning the globus pallidus are correct EXCEPT
a)It receives input from the putamen.
b)It receives input from the motor cortex.
c)It projects to ventral anterior nucleus of thalamus.
d)Its medial segment is concordant with pars reticularis of substantia nigra.

14-The hypothalamic nucleus responsible for secretion of releasing factors is


a)Arcuate.
b)Mammillary.
c)Lateral.
d)Paraventricular.
Prof.Adel Kamel 117

15-Fasciculus retroflexus projects from


a)Pineal body.
b)Habenular nucleus.
c)Red nucleus.
d)Inferior colliculus.

Problems
1-A 60 year old patient came to the emergency room with right side paralysis and left
ophthalmoplegia. His left pupil was dilated and fixed. Name the syndrome and explain the
anatomical basis of its manifestations.
2-Using your neuroanatomical information how can you differentiate between:
a)Upper and lower motor neuron paralysis.
b)Resting and intention tremors.
c)Sensory and motor ataxia.
d)Thalamic and hypothalamic syndromes.
e)Conus medullaris and caudal equine syndromes.
3-A 50 year old female came to the ENT clinic suffering from vertigo, left progressive
diminution of hearing, inability to close the left eye and deviation of the angle of the mouth
to the right side. The case was diagnosed as acoustic neuroma (tumor of the cochlear
nerve). What is the possible location of this tumor and how can you explain the anatomical
background of its manifestations.

You might also like